upper limb Flashcards

1
Q

A 45-year-old woman is being examined as a candidate
for cosmetic breast surgery. The surgeon notes
that both of her breasts sag considerably. Which structure
has most likely become stretched to result in this
condition?
⃣ A. Scarpa’s fascia
⃣ B. Pectoralis major muscle
⃣ C. Pectoralis minor muscle
⃣ D. Suspensory (Cooper’s) ligaments
⃣ E. Serratus anterior muscle

A

D. The suspensory ligaments of the breast, also
known as Cooper’s ligaments, are fi brous bands that
run from the dermis of the skin to the deep layer of
superfi cial fascia and are primary supports for the
breasts against gravity. Ptosis of the breast is usually
due to the stretching of these ligaments and can be
repaired with plastic surgery. Scarpa’s fascia is the
deep membranous layer of superfi cial fascia of the
anterior abdominal wall. The pectoralis major and
pectoralis minor are muscles that move the upper
limb and lie deep to the breast but do not provide any
direct support structure to the breast. The serratus
anterior muscle is involved in the movements of the
scapula.
GAS 131, 137-138; GA 62

How well did you know this?
1
Not at all
2
3
4
5
Perfectly
2
Q

A 27-year-old man was admitted to the emergency
department after an automobile collision in which he
suffered a fracture of the lateral border of the scapula.
Six weeks after the accident, physical examination reveals
weakness in medial rotation and adduction of the
humerus. Which nerve was most likely injured?
⃣ A. Lower subscapular
⃣ B. Axillary
⃣ C. Radial
⃣ D. Spinal accessory
⃣ E. Ulnar

A

A. Lower subscapular nerves arise from the cervical
spinal nerves 5 and 6. It innervates the subscapularis
and teres major muscles. The subscapularis and
teres major are both responsible for adducting and
medially rotating the arm. A lesion of this nerve
would result in weakness in these motions. The axillary
nerve also arises from cervical spinal nerves 5
and 6 and innervates the deltoid and teres minor
muscles. The deltoid muscle is large and covers the
entire surface of the shoulder, and contributes to arm
movement in any plane. The teres minor is a lateral
rotator and a member of the rotator cuff group of
muscles. The radial nerve arises from the posterior
cord of the brachial plexus. It is the largest branch,
and it innervates the triceps brachii and anconeus in
the arm. The spinal accessory nerve is cranial nerve
XI, and it innervates the trapezius muscle, which elevates
and depresses the scapula. The ulnar nerve
arises from the medial cord of the brachial plexus and
runs down the medial aspect of the arm. It innervates
muscles of the forearm and hand.
GAS 676-682; GA 366, 369

How well did you know this?
1
Not at all
2
3
4
5
Perfectly
3
Q

A 48-year-old female court stenographer is admitted
to the orthopedic clinic with symptoms of carpal
tunnel syndrome, with which she has suffered for almost
a year. Which muscles most typically become
weakened in this condition?
⃣ A. Dorsal interossei
⃣ B. Lumbricals III and IV
⃣ C. Thenar
⃣ D. Palmar interossei
⃣ E. Hypothenar

A

C. The thenar muscles (and lumbricals I and II) are
innervated by the median nerve, which runs through
the carpal tunnel. The carpal tunnel is formed anteriorly
by the fl exor retinaculum and posteriorly by the
carpal bones. Carpal tunnel syndrome is caused by a
compression of the median nerve, due to reduced
space in the carpal tunnel. The carpal tunnel contains
the tendons of fl exor pollicis longus, fl exor digitorum
profundus, and fl exor digitorum superfi cialis muscles.
The dorsal interossei, lumbricals III and IV, palmar interossei,
and hypothenar muscles are all innervated by
the ulnar nerve.
GAS 756-758, 764-765, 788; GA 8, 400

How well did you know this?
1
Not at all
2
3
4
5
Perfectly
4
Q

A 45-year-old male arrived at the emergency department
with injuries to his left elbow after he fell in
a bicycle race. Radiographic and MRI examinations
show a fracture of the medial epicondyle and a torn
ulnar nerve. Which of the following muscles would be
most likely to be paralyzed?
⃣ A. Flexor digitorum superfi cialis
⃣ B. Biceps brachii
⃣ C. Brachioradialis
⃣ D. Flexor carpi ulnaris
⃣ E. Supinator

A

D. Fracture of the medial epicondyle often causes
damage to the ulnar nerve due to its position in the
groove behind the epicondyle. The ulnar nerve innervates
one and a half muscles in the forearm—the
fl exor carpi ulnaris and the medial half of the fl exor
digitorum profundus. The nerve continues on to innervate
muscles in the hand. The fl exor digitorum
superfi cialis is innervated by the median nerve and
the biceps brachii by the musculocutaneous. The radial
nerve innervates both the brachioradialis and
supinator muscles.
GAS 724-730; GA 378, 390, 397-398

How well did you know this?
1
Not at all
2
3
4
5
Perfectly
5
Q

While walking to his classroom building, a fi rstyear
medical student slipped on the wet pavement and
fell against the curb, injuring his right arm. Radiographic
images showed a midshaft fracture of the humerus.
Which pair of structures was most likely injured
at the fracture site?
⃣ A. Median nerve and brachial artery
⃣ B. Axillary nerve and posterior humeral circum-
fl ex artery
⃣ C. Radial nerve and deep brachial artery
⃣ D. Suprascapular nerve and artery
⃣ E. Long thoracic nerve and lateral thoracic artery

A

C. A midshaft humeral fracture can result in injury
to the radial nerve and deep brachial artery because
they lie in the spiral groove located in the
midshaft. Injury to the median nerve and brachial
artery can be caused by a supracondylar fracture
that occurs by falling on an outstretched hand and
partially fl exed elbow. A fracture of the surgical neck
of the humerus can injure the axillary nerve and
posterior humeral circumfl ex artery. The suprascapular
artery and nerve can be injured in a shoulder
dislocation. The long thoracic nerve and lateral thoracic
artery may be damaged during a mastectomy
procedure.
GAS 713, 724; GA 361

How well did you know this?
1
Not at all
2
3
4
5
Perfectly
6
Q

An 18-year-old male is brought to the emergency
department after an injury while playing rugby. Imaging
reveals a transverse fracture of the humerus about
1 inch proximal to the epicondyles. Which nerve is
most frequently injured by the jagged edges of the broken
bone at this location?
⃣ A. Axillary
⃣ B. Median
⃣ C. Musculocutaneous
⃣ D. Radial
⃣ E. Ulnar

A

B. A supracondylar fracture often results in injury
to the median nerve. The course of the median nerve
is anterolateral, and at the elbow it lies medial to the
brachial artery on the brachialis muscle. The axillary
nerve passes posteriorly through the quadrangular
space, accompanied by the posterior circumfl ex humeral
artery, and winds around the surgical neck of
the humerus. Injury to the surgical neck may damage
the axillary nerve. The musculocutaneous nerve
pierces the coracobrachialis muscle and descends between
the biceps and brachialis muscle. It continues
into the forearm as the lateral antebrachial cutaneous
nerve. The ulnar nerve descends behind the medial
epicondyle in its groove and is easily injured and produces
“funny bone” symptoms.
GAS 724-731; GA 361

How well did you know this?
1
Not at all
2
3
4
5
Perfectly
7
Q

A 52-year-old band director suffered problems in
her right arm several days after strenuous fi eld exercises
for a major athletic tournament. Examination in
the orthopedic clinic reveals wrist drop and weakness
of grasp but normal extension of the elbow joint. There
is no loss of sensation in the affected limb. Which
nerve was most likely affected?
⃣ A. Ulnar
⃣ B. Anterior interosseous
⃣ C. Posterior interosseous
⃣ D. Median
⃣ E. Superfi cial radial

A

C. The radial nerve descends posteriorly between
the long and lateral heads of the triceps and passes
inferolaterally on the back of the humerus between
the medial and lateral heads of the triceps. It eventually
enters the anterior compartment and descends to
enter the cubital fossa, where it divides into superfi -
cial and deep branches. The deep branch of the radial
nerve winds laterally around the radius and runs
between the two heads of the supinator and continues
as the posterior interosseous nerve, innervating
extensor muscles of the forearm. Because this injury
does not result in loss of sensation over the skin of
the upper limb, it is likely that the superfi cial branch
of the radial nerve is not injured. If the radial nerve
were injured very proximally, the woman would not
have extension of her elbow. The branches of the
radial nerve to the triceps arise proximal to where the
nerve runs in the spiral groove. The anterior interosseous
nerve arises from the median nerve and supplies
the fl exor digitorum profundus, fl exor pollicis
longus, and pronator quadratus, none of which seem
to be injured in this example. Injury to the median
nerve causes a characteristic fl attening (atrophy) of
the thenar eminence.
GAS 750; GA 403

How well did you know this?
1
Not at all
2
3
4
5
Perfectly
8
Q

A 32-year-old woman is admitted to the emergency
department after an automobile collision. Radiographic
examination reveals multiple fractures of the
humerus. Flexion and supination of the forearm are
severely weakened. She also has loss of sensation on
the lateral surface of the forearm. Which of the following
nerves has most likely been injured?
⃣ A. Radial
⃣ B. Musculocutaneous
⃣ C. Median
⃣ D. Lateral cord of brachial plexus
⃣ E. Lateral cutaneous nerve of the forearm

A

B. The musculocutaneous nerve supplies the biceps
brachii and brachialis, which are the fl exors of the
forearm at the elbow. The musculocutaneous nerve
continues as the lateral antebrachial cutaneous nerve,
which supplies sensation to the lateral side of the forearm
(with the forearm in the anatomic position). The
biceps brachii is the most powerful supinator muscle.
Injury to this nerve would result in weakness of supination
and forearm fl exion and lateral forearm sensory
loss. Injury to the radial nerve would result in weakened
extension and a characteristic wrist drop. Injury to
the median nerve causes paralysis of fl exor digitorum
superfi cialis and other fl exors in the forearm and results
in a characteristic fl attening of the thenar eminence.
The lateral cord of the brachial plexus gives origin both
to the musculocutaneous and lateral pectoral nerves.
There is no indication of pectoral paralysis or weakness.
Injury to the lateral cord can result in weakened fl exion
and supination in the forearm, and weakened adduction
and medial rotation of the arm. The lateral cutaneous
nerve of the forearm is a branch of the musculocutaneous
nerve and does not supply any motor
innervation. Injury to the musculocutaneous nerve
alone is unusual but can follow penetrating injuries.
GAS 720; GA 361, 369-371, 376, 390

How well did you know this?
1
Not at all
2
3
4
5
Perfectly
9
Q

A 24-year-old medical student was bitten at the
base of her thumb by her dog. The wound became
infected and the infection spread into the radial
bursa. The tendon(s) of which muscle will most
likely be affected?
⃣ A. Flexor digitorum profundus
⃣ B. Flexor digitorum superfi cialis
⃣ C. Flexor pollicis longus
⃣ D. Flexor carpi radialis
⃣ E. Flexor pollicis brevis

A

C. Tenosynovitis can be due to an infection of the
synovial sheaths of the digits. Tenosynovitis in the
thumb may spread through the synovial sheath of
the fl exor pollicis longus tendon, also known as the
radial bursa. The tendons of the fl exor digitorum superfi
cialis and profundus muscles are enveloped in
the common synovial fl exor sheath, or ulnar bursa.
Neither the fl exor carpi radialis nor fl exor pollicis brevis
tendons are contained in synovial fl exor sheaths.
GAS 759-761; GA 397-399

How well did you know this?
1
Not at all
2
3
4
5
Perfectly
10
Q

Laboratory studies in the outpatient clinic on a
24-year-old female included assessment of circulating
blood chemistry. Which of the following arteries is
most likely at risk during venipuncture at the cubital
fossa?
⃣ A. Brachial
⃣ B. Common interosseous
⃣ C. Ulnar
⃣ D. Anterior interosseous
⃣ E. Radial

A

A. The three chief contents of the cubital fossa are
the biceps brachii tendon, brachial artery, and median
nerve (lateral to medial). The common and anterior
interosseous arteries arise distal to the cubital fossa; the
ulnar and radial arteries are the result of the bifurcation
of the brachial artery distal to the cubital fossa.
GAS 729; GA 361, 366-368, 380

How well did you know this?
1
Not at all
2
3
4
5
Perfectly
11
Q

A 22-year-old male is diagnosed with metastatic
malignant melanoma of the skin over the xiphoid process.
Which nodes receive most of the lymph from this
area and are therefore most likely to be involved in
metastasis of the tumor?
⃣ A. Deep inguinal
⃣ B. Vertical group of superfi cial inguinal
⃣ C. Horizontal group of superfi cial inguinal
⃣ D. Axillary
⃣ E. Deep and superfi cial inguinal

A

D. Lymph from the skin of the anterior chest
wall primarily drains to the axillary lymph nodes.
GAS 709; GA 12, 133

How well did you know this?
1
Not at all
2
3
4
5
Perfectly
12
Q

A 49-year-old female who had suffered a myocardial
infarction must undergo a bypass graft procedure
using the internal thoracic artery. Which vessels will
most likely continue to supply blood to the anterior
part of the upper intercostal spaces?
⃣ A. Musculophrenic
⃣ B. Superior epigastric
⃣ C. Posterior intercostal
⃣ D. Lateral thoracic
⃣ E. Thoracodorsal

A

C. The anterior intercostal arteries are 12 small
arteries, two in each of the upper six intercostal
spaces at the upper and lower borders. The upper
artery lying in each space anastomoses with the posterior
intercostal arteries, whereas the lower one usually
joins the collateral branch of the posterior intercostal
artery. The musculophrenic artery supplies the
pericardium, diaphragm, and muscles of the abdominal
wall. It anastomoses with the deep circumfl ex iliac
artery. The superior epigastric artery supplies the diaphragm,
peritoneum, and the anterior abdominal wall
and anastomoses with the inferior epigastric artery.
The lateral thoracic artery runs along the lateral border
of the pectoralis minor muscle and supplies the
pectoralis major, pectoralis minor, and serratus anterior.
The thoracodorsal artery accompanies the thoracodorsal
nerve in supplying the latissimus dorsi
muscle and lateral thoracic wall.
GAS 151-155; GA 68

How well did you know this?
1
Not at all
2
3
4
5
Perfectly
13
Q

A 22-year-old woman is admitted to the emergency
department in an unconscious state. The nurse
takes a radial pulse to determine the heart rate of the
patient. This pulse is felt lateral to which tendon?
⃣ A. Palmaris longus
⃣ B. Flexor pollicis longus
⃣ C. Flexor digitorum profundus
⃣ D. Flexor carpi radialis
⃣ E. Flexor digitorum superfi cialis

A

D. The location for palpation of the radial pulse is
lateral to the tendon of the fl exor carpi radialis, where
the radial artery can be compressed against the distal
radius. The radial pulse can also be felt in the anatomic
snuffbox between the tendons of the extensor pollicis
brevis and extensor pollicis longus muscles, where the
radial artery can be compressed against the scaphoid.
GAS 349-377; GA 374, 390, 397-398

How well did you know this?
1
Not at all
2
3
4
5
Perfectly
14
Q

A 45-year-old male is admitted to the hospital
after accidentally walking through a plate glass door
in a bar while intoxicated. Physical examination
shows multiple lacerations to the upper limb, with inability
to fl ex the distal interphalangeal joints of the
fourth and fi fth digits. Which of the following muscles
is most likely affected?
⃣ A. Flexor digitorum profundus
⃣ B. Flexor digitorum superfi cialis
⃣ C. Lumbricals
⃣ D. Flexor digitorum profundus and fl exor digitorum
superfi cialis
⃣ E. Interossei

A

A. The fl exor digitorum profundus is dually innervated
by the ulnar nerve to the medial phalanges
and the median nerve for the lateral phalanges. Because
of the superfi cial course of the ulnar nerve, it is
vulnerable to laceration. Such an injury would result
in an inability to fl ex the distal interphalangeal joints
of the fourth and fi fth digits. The fl exor digitorum
superfi cialis is innervated by the median nerve only,
and the course of this nerve runs too deep to be affected
by lacerations. The lumbricals function to fl ex
the MP joints and assist in extending the IP joints.
The interossei adduct and abduct the fi ngers.
GAS 736; GA 399, 402

How well did you know this?
1
Not at all
2
3
4
5
Perfectly
15
Q

A 24-year-old man is admitted with a wound to
the palm of his hand. He cannot touch the pads of his
fi ngers with his thumb but can grip a sheet of paper
between all fi ngers and has no loss of sensation on the
skin of his hand. Which of the following nerves has
most likely been injured?
⃣ A. Deep branch of ulnar
⃣ B. Anterior interosseous
⃣ C. Median
⃣ D. Recurrent branch of median
⃣ E. Deep branch of radial

A

D. The recurrent branch of the median nerve is
motor to the muscles of the thenar eminence, which
is an elevation caused by the abductor pollicis brevis,
fl exor pollicis brevis, and opponens pollicis. If the opponens
pollicis is paralyzed, one cannot oppose the
pad of the thumb to the pads of the other digits. The
recurrent branch does not have a cutaneous distribution.
Holding a piece of paper between the fi ngers is
a simple test of adduction of the fi ngers. These movements
are controlled by the deep branch of the ulnar
nerve, which is not injured in this patient.
GAS 770, 773; GA 414, 417

How well did you know this?
1
Not at all
2
3
4
5
Perfectly
16
Q

A 55-year-old male is examined in a neighborhood
clinic after receiving blunt trauma to his right
axilla in a fall. He has diffi culty elevating the right arm
above the level of his shoulder. Physical examination
shows the inferior angle of his right scapula protrudes
more than the lower part of the left scapula. The right
scapula protrudes far more when the patient pushes
against resistance. Which of the following neural structures
has most likely been injured?
⃣ A. The posterior cord of the brachial plexus
⃣ B. The long thoracic nerve
⃣ C. The upper trunk of the brachial plexus
⃣ D. The site of origin of the middle and lower
subscapular nerves
⃣ E. Spinal nerve roots C7, C8, and T1

A

B. “Winging” of the scapula occurs when the medial
border of the scapula lifts off the chest wall when
the patient pushes against resistance, such as a wall.
The serratus anterior muscle holds the medial border of
the scapula against the chest wall and is innervated by
the long thoracic nerve. The serratus anterior assists in
abduction of the arm above the horizontal plane.
GAS 690; GA 361

How well did you know this?
1
Not at all
2
3
4
5
Perfectly
17
Q

A mother tugs violently on her male child’s hand
to pull him out of the way of an oncoming car and the
child screams in pain. Thereafter, it becomes obvious
that the child cannot straighten his forearm at the elbow.
When the child is seen in the emergency department,
radiographic examination reveals a dislocation of the
head of the radius. Which of the following ligaments is
most likely directly associated with this injury?
⃣ A. Anular
⃣ B. Joint capsular
⃣ C. Interosseous
⃣ D. Radial collateral
⃣ E. Ulnar collateral

A

A. The anular ligament is a fi brous band that
encircles the head of the radius, forming a collar that
fuses with the radial collateral ligament and articular
capsule of the elbow. The anular ligament functions
to prevent displacement of the head of the radius
from its socket. The joint capsule functions to allow
free rotation of the joint and does not function in its
stabilization. The interosseous membrane is a fi brous
layer between the radius and ulna helping to hold
these two bones together. The radial collateral ligament
extends from the lateral epicondyle to the margins
of the radial notch of the ulnar and the anular
ligament of the radius. The ulnar collateral ligament
is triangular ligament and extends from the medial
epicondyle to the olecranon of the ulna.
GAS 724-729; GA 388

How well did you know this?
1
Not at all
2
3
4
5
Perfectly
18
Q

After a forceps delivery of a male infant, the baby
presents with his left upper limb adducted, internally
rotated, and fl exed at the wrist. The startle refl ex is absent
on the ipsilateral side. Which part of the brachial
plexus was most likely injured during this delivery?
⃣ A. Lateral cord
⃣ B. Medial cord
⃣ C. Roots of the lower trunk
⃣ D. Root of the middle trunk
⃣ E. Roots of the upper trunk

A

E. The injury being described is also known as
Erb-Duchenne paralysis or “waiter’s tip-hand.” This
usually results from an injury to the upper trunk of the
brachial plexus, presenting with loss of abduction,
fl exion, and lateral rotation of the arm. The superior
trunk of the brachial plexus consists of spinal nerve
roots C5-6.
GAS 700-709; GA 361, 369-371

How well did you know this?
1
Not at all
2
3
4
5
Perfectly
19
Q

A 35-year-old patient has a small but painful tumor
under the nail of the little fi nger. Which of the
following nerves would have to be anesthetized for a
painless removal of the tumor?
⃣ A. Superfi cial radial
⃣ B. Common palmar digital of median
⃣ C. Common palmar digital of ulnar
⃣ D. Deep radial
⃣ E. Recurrent branch of median

A

C. The common palmar digital branch comes off
the superfi cial branch of the ulnar nerve and supplies
the skin of the little fi nger and the medial side of the
ring fi nger. The superfi cial branch of the radial nerve
provides cutaneous innervation to the radial (lateral)
dorsum of the hand and the radial two and a half
digits over the proximal phalanx. The common palmar
digital branch of the median nerve innervates
most of the lateral aspect of the palmar hand and the
dorsal aspect of the second and third fi nger as well as
the lateral part of the fourth digit. The deep radial
nerve supplies the extensor carpi radialis brevis and
supinator muscles and continues as the posterior interosseous
nerve. The recurrent branch of the median
nerve supplies the abductor pollicis brevis, fl exor pollicis
brevis, and opponens pollicis muscles.
GAS 744; GA 414

How well did you know this?
1
Not at all
2
3
4
5
Perfectly
20
Q

A 25-year-old male athlete is admitted to the
emergency department after a bad landing in the pole
vault. Radiographic examination of his hand reveals a
fractured carpal bone in the fl oor of the anatomic
snuffbox ( Fig. 6-1 ). Which bone has most likely been
fractured?
⃣ A. Triquetral
⃣ B. Scaphoid
⃣ C. Capitate
⃣ D. Hamate
⃣ E. Trapezoid

A

B. The anatomic snuffbox is formed by the tendons
of the extensor pollicis brevis, the abductor pollicis
longus, and the extensor pollicis longus. The fl oor
is formed by the scaphoid bone, and it is here that one
can palpate for a possible fractured scaphoid.
GAS 752-754; GA 392, 394, 422

How well did you know this?
1
Not at all
2
3
4
5
Perfectly
21
Q

A 36-year-old man is brought to the emergency
department because of a deep knife wound on the medial
side of his distal forearm. He is unable to hold a
piece of paper between his fi ngers and has sensory loss
on the medial side of his hand and little fi nger. Which
nerve is most likely injured?
⃣ A. Axillary
⃣ B. Median
⃣ C. Musculocutaneous
⃣ D. Radial
⃣ E. Ulnar

A

E. The ulnar nerve innervates the palmar interossei,
which adduct the fi ngers. This is the movement
that would maintain the paper between the fi ngers.
The axillary nerve does not innervate muscles of the
hand. The median nerve supplies the fi rst and second
lumbricals, the opponens pollicis, abductor pollicis
brevis, and the fl exor pollicis brevis. None of these
muscles would affect the ability to hold a piece of paper
between the fi ngers. The musculocutaneous and
radial nerves do not supply muscles of the hand.
GAS 661, 706, 720; GA 369-371, 390, 400, 417

How well did you know this?
1
Not at all
2
3
4
5
Perfectly
22
Q

A 19-year-old man is brought to the emergency
department after dislocating his shoulder while playing
soccer. Following reduction of the dislocation, he has
pain over the dorsal region of the shoulder and cannot
abduct the arm normally. An MRI of the shoulder
shows a torn muscle. Which of the following muscles
is most likely to have been damaged by this injury?
⃣ A. Coracobrachialis
⃣ B. Long head of the triceps
⃣ C. Pectoralis minor
⃣ D. Supraspinatus
⃣ E. Teres major

A

D. The supraspinatus is one of the rotator cuff
muscles. Its tendon is relatively avascular and is often
injured when the shoulder is dislocated. This muscle
initiates abduction of the arm, and damage would
impair this movement. The coracobrachialis muscle,
which runs from the coracoid process to the humerus,
functions in adduction and fl exion of the arm. The
triceps’ main function is to extend the elbow, and
damage to its long head would not affect abduction.
The pectoralis minor functions as an accessory respiratory
muscle and to stabilize the scapula and is not
involved in abduction. The teres major functions to
adduct and medially rotate the arm.
GAS 678-680; GA 38-39, 361, 364

How well did you know this?
1
Not at all
2
3
4
5
Perfectly
23
Q

A 47-year-old female tennis professional is informed
by her physician that she has a rotator cuff injury
that will require surgery. Her physician explains
that over the years of play a shoulder ligament has
gradually caused severe damage to the underlying
muscle. To which of the following ligaments is the physician
most likely referring?
⃣ A. Acromioclavicular ligament
⃣ B. Coracohumeral ligament
⃣ C. Transverse scapular ligament
⃣ D. Glenohumeral ligament
⃣ E. Coracoacromial ligament

A

E. The coracoacromial ligament contributes to
the coracoacromial arch, preventing superior displacement
of the head of the humerus. Because this ligament
is very strong, it will rarely be damaged; instead,
the ligament can cause infl ammation or erosion of the
tendon of the supraspinatus muscle as the tendon
passes back and forth under the ligament. The acromioclavicular
ligament, connecting the acromion with
the lateral end of the clavicle, is not in contact with the
supraspinatus tendon. The coracohumeral ligament is
located too far anteriorly to impinge upon the supraspinatus
tendon. The glenohumeral ligament is located
deep to the rotator cuff muscles and would not contribute
to injury of the supraspinatus muscle. The transverse
scapular ligament crosses the scapular notch and
is not in contact with the supraspinatus tendon.
GAS 665; GA 354, 356

How well did you know this?
1
Not at all
2
3
4
5
Perfectly
24
Q

A 69-year-old man has numbness in the middle
three digits of his right hand and fi nds it diffi cult to
grasp objects with that hand. He states that he retired
9 years earlier, after working as a carpenter for
50 years. He has atrophy of the thenar eminence ( Fig.
6-2 ). Which of the following conditions is the most
likely cause of the problems in his hand?
⃣ A. Compression of the median nerve in the carpal
tunnel
⃣ B. Formation of the osteophytes that compress
the ulnar nerve at the medial epicondyle
⃣ C. Hypertrophy of the triceps muscle compressing
the brachial plexus
⃣ D. Osteoarthritis of the cervical spine
⃣ E. Repeated trauma to the ulnar nerve

A

A. The median nerve supplies sensory innervation
to the thumb, index, and middle fi ngers and also
to the lateral half of the ring fi nger. The median nerve
also provides motor innervation to muscles of the thenar
eminence. Compression of the median nerve in the
carpal tunnel explains these defi cits in conjunction
with normal functioning of the fl exor compartment of
the forearm because these muscles are innervated by
the median nerve proximal to the carpal tunnel. The
ulnar nerve is not implicated in these symptoms. It
does not provide sensation to digits 1 to 3. Compression
of the brachial plexus could not be attributed to
pressure from the triceps because this muscle is located
distal to the plexus. In addition, brachial plexus
symptoms would include other upper limb defi cits,
rather than the focal symptoms described in this case.
Osteoarthritis of the cervical spine would also lead to
increasing complexity of symptoms.
GAS 764, 788; GA 406

How well did you know this?
1
Not at all
2
3
4
5
Perfectly
25
Q

A 13-year-old boy is brought to the emergency
department after losing control during a motorbike race
in which he was run over by several of the other racers.
Physical examination reveals several cuts and bruises.
He is unable to extend the left wrist, fi ngers, and
thumb, although he can extend the elbow. Sensation is
lost in the lateral half of the dorsum of the left hand.
Which of the following nerves has most likely been
injured to result in these signs, and in what part of the
arm is the injury located?
⃣ A. Median nerve, anterior wrist
⃣ B. Median nerve, arm
⃣ C. Radial nerve, midhumerus
⃣ D. Ulnar nerve, midlateral forearm
⃣ E. Ulnar nerve, midpalmar region

A

C. The radial nerve innervates the extensor compartments
of the arm and the forearm. It supplies the
triceps brachii proximal to the spiral groove, so elbow
extension is intact here. It also provides sensory innervation to much of the posterior arm and forearm
as well as the dorsal thumb, index, and middle fi ngers
up to the level of the fi ngernails. Symptoms are described
only in the distal limb due to the midhumeral
location of the lesion. The median nerve innervates
fl exors of the forearm and thenar muscles and provides
sensory innervation to the lateral palmar hand.
The ulnar nerve supplies only the fl exor carpi ulnaris
and the medial half of the fl exor digitorum profundus
in the forearm. Additionally, its sensory distribution is
to both the palmar and dorsal aspects of the medial
hand. It does not supply extensor muscles.
GAS 713, 724; GA 361

How well did you know this?
1
Not at all
2
3
4
5
Perfectly
26
Q

A 17-year-old male has weakness of elbow fl exion
and supination of the left hand after sustaining a knife
wound in that arm in a street fi ght. Examination in the
emergency department indicates that a nerve has been
severed. Which of the following conditions will also
most likely be seen during physical examination?
⃣ A. Inability to adduct and abduct his fi ngers
⃣ B. Inability to fl ex his fi ngers
⃣ C. Inability to fl ex his thumb
⃣ D. Sensory loss over the lateral surface of his
forearm
⃣ E. Sensory loss over the medial surface of his
forearm

A

D. The musculocutaneous nerve innervates the
brachialis and biceps brachii muscles, which are the
main fl exors at the elbow. The biceps inserts on
the radius and is an important supinator. Because the
musculocutaneous nerve is damaged in this case, it
leads to loss of sensory perception to the lateral forearm,
which is supplied by the distal portion of the
musculocutaneous nerve (known as the lateral antebrachial
cutaneous nerve). Adduction and abduction of
the fi ngers are mediated by the ulnar nerve and would
not be affected in this instance. The fl exor pollicis brevis
fl exes the thumb and is mainly innervated by the
recurrent branch of the median nerve. Flexion of the
fi ngers is performed by the long fl exors of the fi ngers
and lumbrical muscles, innervated by the median and
ulnar nerves. Sensory innervation of the medial forearm
is provided by the medial antebrachial cutaneous nerve,
a branch of the medial cord of the brachial plexus.
GAS 744; GA 424-425

How well did you know this?
1
Not at all
2
3
4
5
Perfectly
27
Q

Following several days of 12-hour daily rehearsals
of the symphony orchestra for a performance of a Wagnerian
opera, the 52-year-old male conductor experienced
such excruciating pain in the posterior aspect of
his right forearm that he could no longer direct the
musicians. When the maestro’s forearm was palpated
2 cm distal to, and posteromedial to, the lateral epicondyle,
the resulting excruciating pain caused the conductor
to weep. Injections of steroids and rest were
recommended to ease the pain. Which of the following
injuries is most likely?
⃣ A. Compression of the ulnar nerve by the fl exor
carpi ulnaris
⃣ B. Compression of the median nerve by the pronator
teres
⃣ C. Compression of the median nerve by the
fl exor digitorum superfi cialis
⃣ D. Compression of the superfi cial radial nerve by
the brachioradialis
⃣ E. Compression of the deep radial nerve by the
supinator

A

E. The deep radial nerve courses between the two
heads of the supinator and is located just medial and
distal to the lateral epicondyle. It can be irritated by
hypertrophy of the supinator, which compresses the
nerve, causing pain and weakness. The ulnar nerve
courses laterally behind the medial epicondyle and continues
anterior to the fl exor carpi ulnaris. The median
nerve passes into the forearm fl exor compartment; the
superfi cial radial nerve courses down the lateral aspect
of the posterior forearm and would not cause pain due
to pressure applied to the posterior forearm.
GAS 747; GA 390

How well did you know this?
1
Not at all
2
3
4
5
Perfectly
28
Q

A 54-year-old female marathon runner presents
with pain in her right wrist that resulted when she fell
with force on her outstretched hand. Radiographic
studies indicate an anterior dislocation of a carpal bone
( Fig. 6-3 ). Which of the following bones is most likely
dislocated?
⃣ A. Capitate
⃣ B. Lunate
⃣ C. Scaphoid
⃣ D. Trapezoid
⃣ E. Triquetrum

A

B. The lunate is the most commonly dislocated
carpal bone because of its shape and relatively weak
ligaments anteriorly. Dislocations of the scaphoid and
triquetrum are relatively rare. The trapezoid and capitate
bones are located in the distal row of the carpal
bones.
GAS 752-754; GA 392, 394, 422

How well did you know this?
1
Not at all
2
3
4
5
Perfectly
29
Q

A 45-year-old man is admitted to the hospital after
a car crash. Radiographic examination reveals mild
disk herniations of C7, C8, and T1. The patient presents
with a sensory defi cit of the C8 and T1 spinal
nerve dermatomes. The dorsal root ganglia of C8 and
T1 would contain cell bodies of sensory fi bers carried
by which of the following nerves?
⃣ A. Medial antebrachial cutaneous nerve
⃣ B. Long thoracic nerve
⃣ C. Lateral antebrachial cutaneous nerve
⃣ D. Deep branch of ulnar nerve
⃣ E. Anterior interosseous nerve

A

A. The medial antebrachial cutaneous nerve
carries sensory fi bers derived from the C8 and T1
levels. The lateral antebrachial cutaneous nerve is the
distal continuation of the musculocutaneous nerve,
carrying fi bers from the C5, C6, and C7 levels. The
deep branch of the ulnar nerve and the anterior interosseous
nerves carry predominantly motor fi bers. The
sensory fi bers coursing in the radial nerve are derived
from the C5 to C8 levels.
GAS 700-709; GA 361, 369-371

How well did you know this?
1
Not at all
2
3
4
5
Perfectly
30
Q

A 23-year-old female maid was making a bed in a
hotel bedroom. As she straightened the sheet by running
her right hand over the surface with her fi ngers
extended, she caught the end of the index fi nger in a
fold. She experienced a sudden, severe pain over the
base of the terminal phalanx. Several hours later when
the pain had diminished, she noted that the end of her
right index fi nger was swollen and she could not completely
extend the terminal interphalangeal joint. Which
one of the following structures within the digit was
most likely injured?
⃣ A. The proper palmar digital branch of the median
nerve
⃣ B. The vinculum longa
⃣ C. The insertion of the tendon of the extensor
digitorum onto the base of the distal phalanx
⃣ D. The insertion of the fl exor digitorum profundus
tendon
⃣ E. The insertion of the fl exor digitorum superfi -
cialis tendon

A

C. The contraction of the extensor mechanism
produces extension of the distal interphalangeal joint.
When it is torn from the distal phalanx, the digit is
pulled into fl exion by the fl exor digitorum profundus.
The proper palmar digital branches of the median
nerve supply lumbrical muscles and carry sensation
from their respective digits. Vincula longa are slender,
bandlike connections from the deep fl exor tendons to
the phalanx that can carry blood supply to the tendons.
The insertions of the fl exor digitorum superfi -
cialis and profundus are on the fl exor surface of the
middle and distal phalanges, respectively, and act to
fl ex the interphalangeal joints.
GAS 745-747; GA 378, 401

How well did you know this?
1
Not at all
2
3
4
5
Perfectly
31
Q

A 45-year-old patient had fallen upon his outstretched
hand, resulting in a Smith fracture of the
distal end of the radius. The fractured bone displaced a
carpal bone in the palmar direction, resulting in nerve
compression within the carpal tunnel. Which of the
following carpal bones will most likely be dislocated?
⃣ A. Scaphoid
⃣ B. Trapezium
⃣ C. Capitate
⃣ D. Hamate
⃣ E. Lunate

A

E. In a Smith fracture, the distal fragment of the
radius deviates palmarward, often displacing the lunate
bone. The other listed bones are unlikely to be
displaced in a palmar direction by a Smith fracture.
GAS 752-754; GA 392, 394, 422

How well did you know this?
1
Not at all
2
3
4
5
Perfectly
32
Q

A 15-year-old girl was brought to the emergency
department with a tear of the tendons in the fi rst dorsal
compartment of the wrist from a severe bite by a pit
bull dog. The injured tendons in this compartment
would include which of the following muscles?
⃣ A. Extensor carpi radialis longus and brevis
⃣ B. Abductor pollicis longus and extensor pollicis
brevis
⃣ C. Extensor digitorum
⃣ D. Extensor indicis proprius
⃣ E. Extensor carpi ulnaris

A

B. The abductor pollicis longus and extensor
pollicis brevis are the occupants of the fi rst dorsal
compartment of the wrist. The extensor carpi radialis
longus and brevis are in the second compartment.
The extensor digitorum is in the third compartment,
as is the extensor indicis proprius. The extensor carpi
ulnaris is located in the sixth dorsal compartment.
GAS 748-749; GA 401-402, 412-413

How well did you know this?
1
Not at all
2
3
4
5
Perfectly
33
Q

As she fell from the uneven parallel bars, the 17-
year-old female gymnast grasped the lower bar briefl y
with one hand but then fell painfully to the fl oor. An
MRI examination reveals an injury to the medial cord
of the brachial plexus. Which of the following spinal
nerve levels would most likely be affected?
⃣ A. C5, C6
⃣ B. C6, C7
⃣ C. C7, C8
⃣ D. C7, C8, T1
⃣ E. C8, T1

A

E. The medial cord has been injured by traction
on the lower trunk of the brachial plexus. The medial
cord is the continuation of the inferior (lower) trunk
of the brachial plexus, which is formed by C8 and T1.
C5 and C6 are typically associated with the superior
(upper) trunk level and thus the lateral cord. C7 forms
the middle trunk. An injury to the posterior cord
would usually involve the C7 spinal nerve. This is a
typical Klumpke paralysis.
GAS 700-709; GA 361, 369-371

How well did you know this?
1
Not at all
2
3
4
5
Perfectly
34
Q

A 21-year-old female softball pitcher is examined
in the emergency department after she was struck in
the arm by a line drive (a ball hit very hard and low).
Radiographic and MRI studies show soft tissue injury
to the region of the spiral groove, with trauma to the
radial nerve. Which of the following muscles would be
intact after this injury?
⃣ A. Flexor carpi ulnaris
⃣ B. Extensor indicis
⃣ C. Brachioradialis
⃣ D. Extensor carpi radialis
⃣ E. Supinator

A

A. The fl exor carpi ulnaris muscle is not innervated
by the radial nerve but rather by the ulnar
UPPER LIMB
195
nerve. The brachioradialis, extensor carpi radialis,
and supinator muscles are all innervated by the radial
nerve distal to the spiral groove.
GAS 737-739; GA 378, 390, 397, 398

How well did you know this?
1
Not at all
2
3
4
5
Perfectly
35
Q

Examination of a 21-year-old female athlete with
an injury of the radial nerve in the spiral groove would
typically demonstrate which of the following physical
signs?
⃣ A. Weakness of thumb abduction and thumb
extension
⃣ B. Weakness of thumb opposition
⃣ C. Inability to extend the elbow
⃣ D. Paralysis of pronation of the hand
⃣ E. Paralysis of abduction and adduction of
the arm

A

A. Injury to the radial nerve in the spiral groove
will paralyze the abductor pollicis longus and both
extensors of the thumb. This injury will also lead to
wrist drop (inability to extend the wrist). Weakness of
grip would also occur, although this is not mentioned
in the question. If the wrist is fl exed, fi nger fl exion
and grip strength are weakened because the long
fl exor tendons are not under tension. Note how much
your strength of grip is increased when your wrist is
extended versus when it is fl exed.
GAS 661, 709, 722-724; GA 366, 371

How well did you know this?
1
Not at all
2
3
4
5
Perfectly
36
Q

The 58-year-old convenience store operator had
received a superfi cial bullet wound to the soft tissues on
the medial side of the elbow in an attempted robbery. A
major nerve was repaired at the site where it passed
behind the medial epicondyle. Bleeding was stopped
from an artery that accompanied the nerve in its path
toward the epicondyle. Vascular repair was performed
on this small artery because of its important role in supplying
blood to the nerve. Which of the following arteries
was most likely repaired?
⃣ A. The profunda brachii
⃣ B. The radial collateral artery
⃣ C. The superior ulnar collateral artery
⃣ D. The inferior ulnar collateral artery
⃣ E. The anterior ulnar recurrent artery

A

C. The superior ulnar collateral branch of the
brachial artery accompanies the ulnar nerve in its path
posterior to the medial epicondyle and is important in
the blood supply of the nerve. The profunda brachii
passes down the arm with the radial nerve. The radial
collateral artery arises from the profunda brachii and
anastomoses with the radial recurrent branch of the
radial artery proximal to the elbow laterally. The inferior
ulnar collateral artery arises from the brachial artery
and accompanies the median nerve into the forearm.
The anterior ulnar recurrent artery arises from the
ulnar artery and anastomoses with the inferior ulnar
collateral anterior to the elbow.
GAS 743, 767-769; GA 368

How well did you know this?
1
Not at all
2
3
4
5
Perfectly
37
Q

A 60-year-old male butcher accidentally slashed
his wrist with his butcher knife, partially dividing the
ulnar nerve. Which of the following actions would
most likely be lost as a result of this injury?
⃣ A. Flexion of the proximal interphalangeal joint
of the fi fth digit (little fi nger)
⃣ B. Extension of the thumb
⃣ C. Adduction of the fi fth digit
⃣ D. Abduction of the thumb
⃣ E. Opposition of the thumb

A

C. Adduction of the fi fth digit is produced by
contraction of the third palmar interosseous muscle.
All of the interossei are innervated by the deep branch
of the ulnar nerve. Flexion of the proximal interphalangeal
joint is a function of the fl exor digitorum superfi
cialis, supplied by the median nerve. Opposition
of the thumb is a function of the opponens pollicis,
supplied by the recurrent branch of the median
nerve.
GAS 729, 772-774; GA 414

How well did you know this?
1
Not at all
2
3
4
5
Perfectly
38
Q

A 23-year-old male medical student fell asleep in
his chair with Netter’s Atlas wedged into his axilla.
When he awoke in the morning, he was unable to extend
the forearm, wrist, or fi ngers. Movements of the
ipsilateral shoulder joint appear to be normal. Which of
UPPER LIMB
175
the following nerves was most likely compressed, producing
the symptoms described?
⃣ A. Lateral cord of the brachial plexus
⃣ B. Medial cord of the brachial plexus
⃣ C. Radial nerve
⃣ D. Median nerve
⃣ E. Lateral and medial pectoral nerves

A

C. The radial nerve is the most likely nerve compressed
to cause these symptoms. This type of nerve
palsy is often called “Saturday night palsy.” One reason
for this nickname is that people would supposedly
fall asleep after being intoxicated on a Saturday
night with their arm over the back of a chair, thereby
compressing the nerve in the spiral groove. The radial
nerve innervates all of the extensors of the elbow,
wrist, and fi ngers. Paralysis of the lateral cord of the
brachial plexus would result in loss of the musculocutaneous
nerve and the pectoral nerves, which do not
mediate extension of the forearm or hand. The medial
cord of the brachial plexus branches into the median
nerve and ulnar nerve. Neither of these nerves innervates
muscles that control extension. The median
nerve innervates fl exors of the forearm and the thenar
muscles. The lateral and median pectoral nerves do
not extend into the arm and innervate the pectoralis
major and minor muscles.
GAS 789-790; GA 366, 371

How well did you know this?
1
Not at all
2
3
4
5
Perfectly
39
Q

The fact that the kidneys of a 32-year-old female
patient were failing required that she be placed on dialysis.
However, the search in her upper limb for a
suitable vein was unexpectedly diffi cult. The major
vein on the lateral side of the arm was too small; others
were too delicate. Finally, a vein was found on the medial
side of the arm that passed through the superfi cial
and deep fascia to join veins beside the brachial artery.
Which of the following veins was this?
⃣ A. Basilic
⃣ B. Lateral cubital
⃣ C. Cephalic
⃣ D. Medial cubital
⃣ E. Medial antebrachial

A

A. The basilic vein can be used for dialysis, especially
when the cephalic vein is judged to be too
small, as in this case. The basilic vein can be elevated
from its position as it passes through the fascia on the
medial side of the arm. The cephalic vein passes more
laterally up the limb. The lateral cubital vein is a
tributary to the cephalic vein, and the medial cubital
vein joins the basilic vein—both rather superfi cial in
position. The medial antebrachial vein courses up the
midline of the forearm (antebrachium) ventrally.
GAS 663, 697, 720, 770; GA 350, 360, 377

How well did you know this?
1
Not at all
2
3
4
5
Perfectly
40
Q

A 29-year-old female had sustained a deep laceration
in the proximal part of the forearm. After the
wound is closed, the following functional defi cits are
observed by the neurologist on the service: The fi rst
three digits are in a position of extension and cannot be
fl exed. Digits 4 and 5 are partially fl exed at the metacarpophalangeal
joints and noticeably more fl exed at
the distal interphalangeal joints. Sensation is absent in
the lateral side of the palm and the palmar surfaces of
digits 1 to 3 and half of the fourth digit. Which of the
following nerves has (have) most likely been injured?
⃣ A. Median nerve
⃣ B. Ulnar and median nerve
⃣ C. Ulnar nerve
⃣ D. Radial and ulnar nerve
⃣ E. Radial nerve

A

A. The patient exhibits the classic “benediction
attitude” of the thumb and fi ngers from injury to the
median nerve proximally in the forearm. The thumb
is somewhat extended (radial supplied abductor and
extensors unopposed); digits 2 and 3 are extended
(by intact interossei); digits 4 and 5 are partially
fl exed (by their intact fl exor digitorum profundus). A
lesion of the median nerve would result in weakened
fl exion of the PIP joints of all digits (fl exor digitorum
superfi cialis), loss of fl exion of the interphalangeal
joint of the thumb, the DIP joints of digits 2 and 3
(fl exor digitorum profundus), and weakened fl exion
of the metacarpophalangeal joints of the second and
third digits (fi rst and second lumbricals). A lesion of
both the ulnar and median nerves would cause
weakness or paralysis of fl exion of all of the digits.
A lesion of the ulnar nerve would mostly cause
weakness in fl exion of the DIP of the fourth and fi fth
digits and would affect all of the interosseous muscles
and the lumbricals of the third and fourth digits.
A lesion of the radial nerve would cause weakness
in extension of the wrist, thumb, and metacarpophalangeal
joints.
GAS 724, 789-790; GA 371, 376

How well did you know this?
1
Not at all
2
3
4
5
Perfectly
41
Q

A 35-year-old male wrestler is admitted to the
emergency department with excruciating pain in his
right shoulder and proximal arm. During physical examination
the patient clutches the arm at the elbow with
his opposite hand and is unable to move the injured
limb. Radiographic studies show that the patient has a
dislocation of the humerus at the glenohumeral joint.
Which of the following conditions is the most likely?
⃣ A. The head of the humerus is displaced
anteriorly.
⃣ B. The head of the humerus is displaced
posteriorly.
⃣ C. The head of the humerus is displaced
inferiorly.
⃣ D. The head of the humerus is displaced
superiorly.
⃣ E. The head of the humerus is displaced
medially.

A

C. The head of the humerus is displaced inferiorly
because in that location it is not supported by
rotator cuff muscle tendons or the coracoacromial
arch. It is also pulled anteriorly beneath the coracoid
process by pectoralis and subscapularis muscles. It
would not be displaced posteriorly because it is supported
by the teres minor and infraspinatus muscle
tendons. It would not be displaced superiorly because
the acromioclavicular ligament and supraspinatus reinforce
in that direction. A medial dislocation is
blocked by the subscapularis tendon.
GAS 667; GA 355

How well did you know this?
1
Not at all
2
3
4
5
Perfectly
42
Q

The 35-year-old female patient has a hard nodule
about 1 cm in diameter slightly above and lateral to her
right areola. A specifi c dye is injected into the tissue
around the tumor, and an incision is made to expose
the lymphatic vessels draining the area, for the lymphatic
vessels take up the dye—which is visible to the
eye. The vessels can then be traced to surgically expose
the lymph nodes receiving the lymph from the tumor.
Which of the following nodes will most likely fi rst encounter
the lymph from the tumor?
⃣ A. Anterior axillary (pectoral) nodes
⃣ B. Rotter interpectoral nodes
⃣ C. Parasternal nodes along the internal thoracic
artery and vein
⃣ D. Central axillary nodes
⃣ E. Apical or infraclavicular nodes

A

A. The anterior axillary (or anterior pectoral)
nodes are the fi rst lymph nodes to receive most of the
lymph from the breast parenchyma, areola, and nipple.
From there, lymph fl ows through central axillary,
apical, and supraclavicular nodes in sequence. Rotter’s
nodes lie between the pectoral muscles and are,
unfortunately, an alternate route in some patients,
speeding the rate of metastasis. The parasternal nodes
receive lymph from the medial part of the breast and
lie along the internal thoracic artery and vein.
GAS 709; GA 63

How well did you know this?
1
Not at all
2
3
4
5
Perfectly
43
Q

During a fi ght in a tavern, a 45-year-old male construction
worker received a shallow stab wound from a
broken beer bottle at a point near the middle of the left
posterior triangle of his neck. Upon physical examination
it is observed that the left shoulder is drooping
lower than the right shoulder, and the superior angle of
the scapula juts out slightly. Strength in turning the head
to the right or left appears to be symmetric. Which of the
following nerves is most likely injured?
⃣ A. Suprascapular nerve in the supraspinous fossa
⃣ B. The terminal segment of the dorsal scapular
nerve
⃣ C. The upper trunk of the brachial plexus
⃣ D. The spinal accessory nerve in the posterior
cervical triangle
⃣ E. The thoracodorsal nerve in the axilla

A

D. The left spinal accessory nerve has been injured
distal to the sternocleidomastoid muscle, resulting
in paralysis of the trapezius, allowing the shoulder
to droop and the superior angle to push out posteriorly.
The sternocleidomastoid muscles are intact, as
demonstrated by symmetry in strength in turning the
head to the right and left. There is no indication of
paralysis of the lateral rotators of the shoulder or elbow
fl exors (suprascapular nerve or upper trunk).
Thoracodorsal nerve injury would result in paralysis
of the latissimus dorsi, an extensor, and medial rotator
of the humerus.
GAS 667, 973-974; GA 370

How well did you know this?
1
Not at all
2
3
4
5
Perfectly
44
Q

A 44-year-old woman is diagnosed with radial
nerve palsy. When muscle function is examined at the
metacarpophalangeal (MCP), proximal interphalangeal
(PIP), and distal interphalangeal (DIP) joints, what
fi ndings are most likely to be present?
⃣ A. Inability to abduct the digits at the MCP joint
⃣ B. Inability to adduct the digits at the MCP joint
⃣ C. Inability to extend the MCP joints only
⃣ D. Inability to extend the MCP, PIP, and DIP
joints
⃣ E. Inability to extend the PIP and DIP joints

A

C. Inability to extend MCP joints. The tendons
of the extensor digitorum and extensor digiti minimi,
innervated by the radial nerve, are responsible for
extension of the MCP, and to a much lesser degree,
the PIP and DIP joints. Abduction and adduction of
the MCP joints are functions of the interossei, all of
which are innervated by the deep ulnar nerve. Extension
of the PIP and DIP joints is performed by the
lumbricals and interossei. The fi rst two lumbricals
are supplied by the median nerve; the other lumbricals
and the interossei, by the deep branch of the
ulnar nerve.
GAS 655; GA 395

How well did you know this?
1
Not at all
2
3
4
5
Perfectly
45
Q

A 27-year-old male painter is admitted to the hospital
after falling from a ladder. Physical examination
reveals that the patient is unable to abduct his arm
more than 15 ° and cannot rotate the arm laterally. A
radiographic examination reveals an oblique fracture of
the humerus. He has associated sensory loss over the
shoulder area. Which of the following injuries will
most likely correspond to the symptoms of the physical
examination?
⃣ A. Fracture of the medial epicondyle
⃣ B. Fracture of the glenoid fossa
⃣ C. Fracture of the surgical neck of the humerus
⃣ D. Fracture of the anatomic neck of the humerus
⃣ E. Fracture of the middle third of the humerus

A

C. Fracture of the surgical neck of the humerus
often injures the axillary nerve, which innervates the
deltoid and teres minor muscles. Abduction of the
humerus between 15 ° and the horizontal is performed
by the deltoid muscle. Lateral rotation of the
humerus is mainly performed by the deltoid muscle,
teres minor, and the infraspinatus. The deltoid and
teres minor are both lost in this case. Fracture of the
glenoid fossa would lead to drooping of the shoulder.
Fracture of the anatomic neck of the humerus will
similarly lead to a drooping of the shoulder but would
not necessarily affect abduction of the humerus. It is
also quite unusual. Fracture of the middle third of the
humerus would most likely injure the radial nerve.
The ulnar nerve would be potentially compromised in
a fracture of the medial epicondyle of the humerus.
GAS 667, 668; GA 355

How well did you know this?
1
Not at all
2
3
4
5
Perfectly
46
Q

A 47-year-old female patient’s right breast exhibited
characteristics of peau d’orange; that is, the skin
resembled orange peel. This condition is primarily a
result of which of the following?
⃣ A. Shortening of the suspensory ligaments by
cancer in the axillary tail
⃣ B. Blockage of cutaneous lymphatic vessels
⃣ C. Contraction of the retinacula cutis of the areola
and nipple
⃣ D. Invasion of the pectoralis major by the cancer
⃣ E. Ipsilateral (same side) inversion of the periareolar
skin from ductular cancer

A

B. When cutaneous lymphatics of the breast are
blocked by cancer, the skin becomes edematous, except
where hair follicles cause small indentations of
the skin, giving an overall resemblance to orange
peel. Shortening of the suspensory ligaments or retinacula
cutis leads to pitting of the overlying skin,
pitting that is intensifi ed if the patient raises her arm
above her head. Invasion of the pectoralis major by
cancer can result in fi xation of the breast, seen upon
elevation of the ipsilateral limb. Inversion of areolar
skin with involvement of the ducts would also be due
to involvement of the retinacula cutis.
GAS 709; GA 381

How well did you know this?
1
Not at all
2
3
4
5
Perfectly
47
Q

A 29-year-old female is examined in the emergency
department after falling from her balcony. Radiographic
examination reveals that she has suffered a
broken clavicle, with associated internal bleeding.
Which of the following vessels is most likely to be injured
in clavicular fractures?
⃣ A. Subclavian artery
⃣ B. Cephalic vein
⃣ C. Lateral thoracic artery
⃣ D. Subclavian vein
⃣ E. Internal thoracic artery

A

D. The subclavian vein traverses between the
clavicle and fi rst rib and is the most superfi cial structure
to be damaged following a fracture of the clavicle.
The subclavian artery runs deep to the subclavian
vein, and though it is in the appropriate location, it
would likely not be damaged because of its deep anatomic
position. The cephalic vein is a tributary to the
axillary vein after ascending on the lateral side of the
arm. Its location within the body is too superfi cial and
lateral to the site of injury. The lateral thoracic artery
is a branch from the axillary artery that runs lateral to
the pectoralis minor. It courses inferior and medial
from its point of origin from the axillary artery, and it
does not maintain a position near the clavicle during
its descent. The internal thoracic artery arises from
the fi rst part of the subclavian artery before descending
deep to the costal cartilages. Its point of origin
from the subclavian artery is lateral to clavicular injury.
Furthermore, its course behind the costal cartilages
is quite medial to the clavicular fracture.
GAS 697, 952, 961, 973, 978; GA 37

How well did you know this?
1
Not at all
2
3
4
5
Perfectly
48
Q

A 68-year-old female is examined by the senior
resident in emergency medicine after her fall on a wet
bathroom fl oor in the shopping center. Physical examination
reveals a posterior displacement of the left distal
wrist and hand. Radiographic examination reveals an
oblique fracture of the radius. Which of the following
is the most likely fracture involved in this case?
⃣ A. Colles’ fracture
⃣ B. Scaphoid fracture
⃣ C. Bennett’s fracture
⃣ D. Volkmann’s ischemic contracture
⃣ E. Boxer’s fracture

A

A. Colles’ fracture is a fracture of the distal end of
the radius. The proximal portion of the radius is displaced
anteriorly, with the distal bone fragment projecting
posteriorly. The displacement of the radius from the
wrist often gives the appearance of a dinner fork, thus
a Colles’ fracture is often referred to as a “dinner fork”
deformity. A scaphoid fracture results from a fracture of
the scaphoid bone and would thus not cause displace-
ment of the radius. This fracture usually occurs at the
narrow aspect (“waist”) of the scaphoid bone. Bennett’s
and boxer’s fractures both result from fractures of the
metacarpals (fi rst and fi fth, respectively). Volkmann’s
ischemic contracture is a muscular deformity that can
follow a supracondylar fracture of the humerus, with
arterial laceration into the fl exor compartment of the
forearm. Ischemia and muscle contracture, with extreme
pain, accompany this fracture.
GAS 734; GA 392

How well did you know this?
1
Not at all
2
3
4
5
Perfectly
49
Q

A 34-year-old female skier was taken by ambulance
to the hospital after she struck a tree on the ski slope.
Imaging gives evidence of a shoulder separation. Which
of the following typically occurs in this kind of injury?
⃣ A. Displacement of the head of the humerus
from the glenoid cavity
⃣ B. Partial or complete tearing of the coracoclavicular
ligament
⃣ C. Partial or complete tearing of the coracoacromial
ligament
⃣ D. Rupture of the transverse scapular ligament
⃣ E. Disruption of the glenoid labrum

A

B. In shoulder separation, either or both the acromioclavicular
and coracoclavicular ligaments can
be partially or completely torn through. The acromioclavicular
joint can be interrupted and the distal end
of the clavicle may deviate upward in a complete
separation, while the upper limb droops away inferiorly,
causing a “step off” that can be palpated and
sometimes observed. Displacement of the head of the
humerus is shoulder dislocation, not separation. The
coracoacromial ligament is not torn in separation (but
it is sometimes used in the repair of the torn coracoclavicular
ligament). Disruption of the glenoid labrum
often accompanies shoulder dislocation.
GAS 669; GA 354

How well did you know this?
1
Not at all
2
3
4
5
Perfectly
50
Q

A 22-year-old male construction worker is admitted
to the hospital after he suffers a penetrating injury
to his upper limb from a nail gun. Upon physical examination
the patient is unable to fl ex the distal interphalangeal
joints of digits 4 and 5. What is the most
likely cause of his injury?
⃣ A. Trauma to the ulnar nerve near the trochlea
⃣ B. Trauma to the ulnar nerve at the wrist
⃣ C. Median nerve damage proximal to the pronator
teres
⃣ D. Median nerve damage at the wrist
⃣ E. Trauma to spinal nerve root C8

A

A. The nail was fi red explosively from the nail
gun and then pierced the ulnar nerve near the coronoid
process of the ulna trochlea of the humerus.
Paralysis of the medial half of the fl exor digitorum
profundus would result (among other signifi cant defi -
cits), with loss of fl exion of the distal interphalangeal
joints of digits 4 and 5. Ulnar trauma at the wrist
would not affect the interphalangeal joints, although
it would cause paralysis of interossei, hypothenar
muscles, etc. Median nerve damage proximal to the
pronator teres would affect proximal interphalangeal
joint fl exion and distal interphalangeal joint fl exion of
digits 2 and 3 as well as thumb fl exion. Median nerve
injury at the wrist would cause loss of thenar muscles
but not long fl exors of the fi ngers. Trauma to spinal
nerve root C8 would affect all long fi nger fl exors.
GAS 729, 771; GA 376, 390

How well did you know this?
1
Not at all
2
3
4
5
Perfectly
51
Q

The shoulder of a 44-year-old deer hunter had
been penetrated by a bolt released from a crossbow.
The bolt had transected the axillary artery just beyond
the origin of the subscapular artery. A compress is
placed on the wound with deep pressure. After a clamp
is placed on the bleeding artery, thought is given to the
anatomy of the vessel. What collateral arterial pathways
are available to bypass the site of injury?
⃣ A. Suprascapular with circumfl ex scapular artery
⃣ B. Dorsal scapular with thoracodorsal artery
⃣ C. Posterior humeral circumfl ex artery with deep
brachial artery
⃣ D. Lateral thoracic with brachial artery
⃣ E. Supreme thoracic artery with thoracoacromial
artery

A

C. The injury has occurred just beyond the third
part of the axillary artery. The only collateral arterial
channel between the third part of the axillary artery
and the brachial artery is that between the posterior
humeral circumfl ex and the ascending branch of the
profunda brachii—and this anastomotic path is often
inadequate to supply the arterial needs of the limb.
The posterior humeral circumfl ex arises from the
third part of the axillary artery. It typically anastomoses
with a variably small, ascending branch of the
profunda brachii branch of the brachial artery. The
suprascapular artery anastomoses with the circumfl ex
scapular deep to the infraspinatus. The dorsal scapular
artery (passing beneath the medial border of the
scapula) has no anastomosis with thoracodorsal
within the scope of the injury. The lateral thoracic
artery has no anastomoses with the brachial artery.
The supreme thoracic artery (from fi rst part of axillary)
has no helpful anastomoses with the thoracoacromial
(second part of axillary).
GAS 683, 696, 697; GA 368

How well did you know this?
1
Not at all
2
3
4
5
Perfectly
52
Q
A 17-year-old male suffered the most common of
fractures of the carpal bones when he fell on his outstretched
hand. Which bone would this be?
⃣ A. Trapezium
⃣ B. Lunate
UPPER LIMB
177
⃣ C. Pisiform
⃣ D. Hamate
⃣ E. Scaphoid
A

E. The scaphoid (or the older term, navicular)
bone is the most commonly fractured carpal bone.
GAS 752-754; GA 392, 394, 422

How well did you know this?
1
Not at all
2
3
4
5
Perfectly
53
Q

A 54-year-old male cotton farmer visits the outpatient
clinic because of a penetrating injury to his forearm
with a baling hook. After the limb is anesthetized,
the site of the wound is opened and fl ushed thoroughly
to remove all debris. The patient is not able to oppose
the tip of the thumb to the tip of the index fi nger, as in
making the OK sign. He is able to touch the tips of the
ring and little fi ngers to the pad of his thumb. What
nerve has most likely been injured?
⃣ A. Median
⃣ B. Posterior interosseous
⃣ C. Radial
⃣ D. Recurrent median
⃣ E. Anterior interosseous

A

E. The anterior interosseous nerve is a branch of
the median nerve that supplies the fl exor pollicis longus,
the lateral half of the fl exor digitorum profundus,
and the pronator quadratus. If it is injured, fl exion of
the interphalangeal joint of the thumb will be compromised.
The median nerve gives rise to the anterior
interosseous nerve but is not a direct enough answer
as injury to it would result in more widespread effects.
The posterior interosseous nerve supplies extensors
in the forearm, not fl exors. The radial nerve gives
rise to the posterior interosseous nerve and is not associated
with the anterior interosseous nerve; therefore,
it would not have any effect on the fl exors of the
forearm. The recurrent median nerve is also a branch
of the median nerve but supplies the thenar eminence
muscles, and its injury would result in problems with
opposable motion of the thumb.
GAS 743, 751; GA 400

How well did you know this?
1
Not at all
2
3
4
5
Perfectly
54
Q

Endoscopic examination of the shoulder of a
62-year-old female clearly demonstrated erosion of the
tendon within the glenohumeral joint. What tendon
was this?
⃣ A. Glenohumeral
⃣ B. Long head of triceps
⃣ C. Long head of biceps
⃣ D. Infraspinatus
⃣ E. Coracobrachialis

A

C. The tendon of the long head of the biceps
brachii muscle passes through the glenohumeral joint,
surrounded by synovial membrane. The glenohumeral
is a ligament that attaches to the glenoid labrum.
The long head of the triceps arises from the
infraglenoid tubercle, beneath the glenoid fossa. The
infraspinatus tendon passes posterior to the head of
the humerus to insert on the greater tubercle. The
coracobrachialis arises from the coracoid process and
inserts on the humerus.
GAS 694, 715-716, 735; GA 370

How well did you know this?
1
Not at all
2
3
4
5
Perfectly
55
Q

The orthopedic surgeon exposed the muscle in
the supraspinous fossa so that she could move it laterally,
in repair of an injured rotator cuff. As she refl ected
the muscle from its bed, an artery was exposed crossing
the ligament that bridges the notch in the superior
border of the scapula. What artery was this?
⃣ A. Subscapular
⃣ B. Transverse cervical
⃣ C. Dorsal scapular
⃣ D. Posterior humeral circumfl ex
⃣ E. Suprascapular

A

E. The suprascapular artery passes over, and the
suprascapular nerve passes under, the superior transverse
scapular ligament. This ligament bridges the
suprascapular notch in the upper border of the scapula.
The artery and nerve then pass deep to the supraspinatus
muscle, thereafter supplying it and then
passing through the spinoglenoid notch to supply the
infraspinatus. The subscapular artery is a branch of
the third part of the axillary artery; it divides into
circumfl ex scapular and thoracodorsal branches. The
transverse cervical artery courses anterior to this site.
The dorsal scapular artery and nerve pass deep to the
medial border of the scapula. The posterior humeral
circumfl ex branch of the axillary artery passes through
the quadrangular space with the axillary nerve.
GAS 696, 697; GA 366, 368

How well did you know this?
1
Not at all
2
3
4
5
Perfectly
56
Q

A 61-year-old man was hit by the cricket bat in
the midhumeral region of his left arm. Physical examination
reveals an inability to extend the wrist and loss
of sensation on a small area of skin on the dorsum of
the hand proximal to the fi rst two fi ngers. What nerve
supplies this specifi c region of the hand?
⃣ A. Radial
⃣ B. Posterior interosseous
⃣ C. Lateral antebrachial cutaneous
⃣ D. Medial antebrachial cutaneous
⃣ E. Dorsal cutaneous of ulnar

A

A. The patient has suffered injury to the radial
nerve in the midhumeral region. The nerve that provides
sensation to the dorsum of the hand proximal to
the thumb and index fi nger is the superfi cial branch of
the radial nerve. The posterior interosseous nerve supplies
a strip of skin on the back of the forearm and
wrist extensors. The lateral antebrachial cutaneous
nerve is a continuation of the musculocutaneous nerve
and supplies the lateral side of the forearm. The medial
antebrachial cutaneous is a direct branch of the medial
cord and supplies skin of the medial side of the forearm.
The dorsal cutaneous branch of the ulnar nerve
supplies the medial side of the dorsum of the hand.
GAS 772-774; GA 417

How well did you know this?
1
Not at all
2
3
4
5
Perfectly
57
Q

A 45-year-old woman is admitted to the hospital
with neck pain. An MRI examination reveals a herniated
disk in the cervical region. Physical examination
reveals weakness in wrist extension and paraesthesia
on the back of her arm and forearm. Which of the following
spinal nerves is most likely injured?
⃣ A. C5
⃣ B. C6
⃣ C. C7
⃣ D. C8
⃣ E. T1

A

C. The seventh cervical nerve makes a major
contribution to the radial nerve, and this nerve is the
prime mover in wrist extension. The dermatome of C7
is in the region described.
GAS 700-709; GA 361, 369-371

58
Q

A 22-year-old male football player suffered a wrist
injury while falling with force on his outstretched hand.
When the anatomic snuffbox is exposed in surgery, an
artery is visualized crossing the fractured bone that provides
a fl oor for this space. What artery was visualized?
⃣ A. Ulnar
⃣ B. Radial
⃣ C. Anterior interosseous
⃣ D. Posterior interosseous
⃣ E. Deep palmar arch

A

B. As the radial artery passes from the ventral
surface of the wrist to the dorsum, it crosses through
the anatomic snuffbox, passing over the scaphoid
bone. The ulnar artery at the wrist is located on the
medial side of the wrist, passing from beneath
the fl exor carpi ulnaris to reach Guyon’s canal between
the pisiform bone and the fl exor retinaculum.
The anterior interosseous and posterior interosseous
arteries arise from the common interosseous branch
of the ulnar artery and pass proximal to distal in the
forearm between the radius and ulna, in the fl exor
and extensor compartments, respectively. The deep
palmar branch of the ulnar artery passes between the
two heads of the adductor pollicis to anastomose with
the radial artery in the palm.
GAS 759; GA 422

59
Q

The right shoulder of a 78-year-old female had
become increasingly painful over the past year. Abduction
of the right arm caused her to wince from the
discomfort. Palpation of the deltoid muscle by the physician
produced exquisite pain. Imaging studies reveal
intermuscular infl ammation extending over the head of
the humerus. What structure was infl amed?
⃣ A. Subscapular bursa
⃣ B. Infraspinatus muscle
⃣ C. Glenohumeral joint cavity
⃣ D. Subacromial bursa
⃣ E. Teres minor muscle

A

D. The patient is suffering from subacromial or
subdeltoid bursitis. (If the pain on palpation is less
when the arm has been elevated to the horizontal, the
bursitis may be thought of as being more subacromial,
that is, associated more with the supraspinatus tendon
perhaps, for such a bursa may be drawn back under
the acromion when the limb is abducted.) The subscapular
bursa, beneath the subscapularis muscle,
would not present as superfi cial pain. It can communicate
with the glenohumeral joint cavity. Infl ammation
or arthritic changes within the glenohumeral joint present
as more generalized shoulder pain than that present
here. The teres minor muscle and tendon are located
inferior to the point of marked discomfort.
GAS 670; GA 358

60
Q

A 55-year-old male metallurgist had been diagnosed
with carpal tunnel syndrome. To begin the operation,
an anesthetic injection into his axillary sheath
was used instead of a general anesthesia. From which
of the following structures does the axillary sheath take
origin?
⃣ A. Superfi cial fascia of the neck
⃣ B. Superfi cial cervical investing fascia
⃣ C. Buccopharyngeal fascia
⃣ D. Clavipectoral fascia
⃣ E. Prevertebral fascia

A

E. The axillary sheath is a fascial continuation
of the prevertebral layer of the deep cervical fascia
extending into the axilla. It encloses the nerves of the
neurovascular bundle of the upper limb. Superfi cial
fascia is loose connective tissue between the dermis
and the deep investing fascia and contains fat, cutaneous
vessels, nerves, lymphatics, and glands. The
buccopharyngeal fascia covers the buccinator muscles
and the pharynx mingles with the pretrachial
fascia. The clavipectoral muscle invests the clavicle
and pectoralis minor muscle. The axillary fascia is
continuous with the pectoral and latissimus dorsi
fascia and forms the hollow of the armpit.
GAS 700-709; GA 361, 369-371

61
Q

A 45-year-old woman is admitted to the hospital
with neck pain. A CT scan reveals a tumor in the left
side of her oral cavity. The tumor and related tissues
are removed with a radical neck surgical procedure.
Two months postoperatively the patient’s left shoulder
droops quite noticeably. Physical examination reveals
distinct weakness in turning her head to the right and
impairment of abduction of her left upper limb to
the level of the shoulder. Which of the following structures
was most likely injured during the radical neck
surgery?
⃣ A. Suprascapular nerve
⃣ B. Long thoracic nerve
⃣ C. Spinal accessory nerve
⃣ D. The junction of spinal nerves C5 and C6 of
the brachial plexus
⃣ E. Radial nerve

A

C. The spinal accessory nerve arises from the
ventral rootlets of C1 to C4 and ascends through the
foramen magnum to then exit the cranial cavity
through the jugular foramen. It innervates the sternocleidomastoid
and trapezius muscles, which function
in head rotation and raising of the shoulders. The suprascapular
nerve receives fi bers from C5-6 (occasionally
from C4 if the plexus is “prefi xed”) and innervates
the supraspinatus muscle, which is responsible for the
fi rst 15 ° of arm abduction. Erb point of the brachial
plexus is at the union of C5-6 spinal nerves. The long
thoracic nerve arises from plexus routes C5, 6, and 7,
and supplies the serratus anterior.
GAS 700-706; GA 361, 369-371

62
Q

A 23-year-old male basketball player is admitted
to the hospital after injuring his shoulder during a
game. Physical and radiographic examinations reveal
total separation of the shoulder ( Fig. 6-4 ). Which of the
following structures has most likely been torn?
⃣ A. Glenohumeral ligament
⃣ B. Coracoacromial ligament
⃣ C. Tendon of long head of biceps brachii
⃣ D. Acromioclavicular ligament
⃣ E. Transverse scapular ligament

A

D. The acromioclavicular ligament connects the
clavicle to the coracoid process of the scapula. Separation
of the shoulder (dislocation of the acromioclavicular
[AC] joint) is associated with damage to the
acromioclavicar ligament (capsule of the AC joint)
and, in more severe injuries, disruption of the coracoclavicular
ligaments (conoid and trapezoid portions).
The glenohumeral ligament may be injured by an anterior
dislocation of the humerus but is not likely to be
injured by a separated shoulder. The coracoacromial
ligament, transverse scapular ligament, and tendon of
the long head of triceps brachii are not likely to be
injured by separation of the shoulder.
GAS 669; GA 354, 363

63
Q

A 35-year-old male body builder has enlarged his
shoulder muscles to such a degree that the size of the
quadrangular space is greatly reduced. Which of the
following structures would most likely be compressed
in this condition?
⃣ A. Axillary nerve
⃣ B. Anterior humeral circumfl ex artery
⃣ C. Cephalic vein
⃣ D. Radial nerve
⃣ E. Subscapular artery

A

A. The quadrangular space is bordered medially
by the long head of the triceps, laterally by the surgical
neck of the humerus, superiorly by the teres minor
and subscapularis muscles, and inferiorly by the teres
major muscle. Both the axillary nerve and posterior
humeral circumfl ex vessels traverse this space. The
other structures listed are not contained within the
quadrangular space. The cephalic vein is located in
the deltopectoral triangle, and the radial nerve is located
in the triangular interval.
GAS 700-709; GA 361, 369-371

64
Q

A 43-year-old woman visits the outpatient clinic
with a neurologic problem. Diagnostically, she cannot
hold a piece of paper between her thumb and the lateral
side of her index fi nger without fl exing the distal
joint of her thumb. This is a positive Froment sign and
a diagnosis of ulnar neuropathy. Weakness of which
specifi c muscle causes this sign to appear?
⃣ A. Flexor pollicis longus
⃣ B. Adductor pollicis
⃣ C. Flexor digiti minimi
⃣ D. Flexor carpi radialis
⃣ E. Extensor indicis

A

B. Froment’s sign is positive for ulnar nerve
palsy. More specifi cally it tests the action of the adductor
pollicis muscle. The patient is asked to hold
a sheet of paper between the thumb and a fl at palm.
The fl exor pollicis longus is innervated by the anterior
interosseous branch of the median nerve. The
fl exor digiti minimi is innervated by the deep branch
of the ulnar nerve and would not be used to hold a
sheet of paper between the thumb and palm. The
fl exor carpi radialis is innervated by the median
nerve, and the extensor indicis is innervated by the
radial nerve.
GAS 763-765; GA 410, 412

65
Q

A 48-year-old female piano player visited the
outpatient clinic with numbness and tingling in her
left hand. A diagnosis was made of nerve compression
in the carpal tunnel, and the patient underwent
an endoscopic nerve release. Two weeks postoperatively
the patient complained of a profound weakness
in the thumb, with loss of thumb opposition. The
sensation to the hand, however, was unaffected.
Which of the following nerves was injured during the
operation?
⃣ A. The fi rst common digital branch of the median
nerve
⃣ B. The second common digital branch of the median
nerve
⃣ C. Recurrent branch of median nerve
⃣ D. Deep branch of the ulnar nerve
⃣ E. Anterior interosseus nerve

A

C. The recurrent branch of the median nerve innervates
the thenar muscles (opponens pollicis, abductor
pollicis brevis, and fl exor pollicis brevis) and is
not responsible for any cutaneous innervation. Damage
to the palmar cutaneous branches of the median
nerve or to the ulnar nerve would not cause weakness
of opposition of the thumb for they are principally
sensory in function. The deep branch of the ulnar
nerve supplies the hypothenar muscles, adductor and
abductor muscles of digits 2–5, and does not innervate
the abductor policis brevis.
GAS 770-771; GA 414

66
Q

A 19-year-old male had suffered a deep laceration
to an upper limb when he stumbled and fell on a
broken bottle. On examination of hand function it is
observed that he is able to extend the metacarpophalangeal
joints of all his fi ngers in the affected limb. He
cannot extend the interphalangeal joints of the fourth
and fi fth digits, and extension of the interphalangeal
joints of the second and third digits is very weak. There is no apparent sensory defi cit in the hand.
Which of the following nerves has most likely been
injured?
⃣ A. Radial nerve at the elbow
⃣ B. Median nerve at the wrist
⃣ C. Ulnar nerve in midforearm
⃣ D. Deep branch of ulnar nerve
⃣ E. Recurrent branch of the median nerve

A

D. Injury to the deep branch of the ulnar nerve
results in paralysis of all interosseous muscles and
the lumbrical muscles of digits 4 and 5. Extension of
the metacarpophalangeal joints is intact, a function
of the radial nerve. Interphalangeal extension of
digits 4 and 5 is absent, due to the loss of all interosseous
muscle and the lumbricals of digits 4 and 5.
Some weak interphalangeal joint extension is still
present in digits 2 and 3 because the lumbricals of
these two fi ngers are innervated by the median
nerve. The radial nerve and the median nerve appear
to be intact in this case. If the ulnar nerve were
injured in the midforearm region, there would be
sensory loss in the palm and digits 4 and 5 and on
the dorsum of the hand. The recurrent branch of the
median nerve supplies the thenar muscles; it does
not supply lumbricals. Moreover, paralysis of this
nerve would have no effect on the interphalangeal
joints.
GAS 770-771; GA 415

67
Q

A 41-year-old woman is scheduled for a latissimus
dorsi muscle fl ap to cosmetically augment the site
of her absent left breast, postmastectomy. Part of the
latissimus dorsi muscle is advanced to the anterior
thoracic wall, based upon arterial supply provided in
part by the artery that passes through the triangular
space of the axilla. What artery is forming the vascular
base of this fl ap?
⃣ A. Circumfl ex scapular artery
⃣ B. Dorsal scapular artery
⃣ C. Transverse cervical artery
⃣ D. Lateral thoracic artery
⃣ E. Thoracoacromial artery

A

A. The circumfl ex scapular artery passes through
the triangular space after arising from the subscapular
artery. It provides superfi cial branches to the overlying
latissimus dorsi, whereas its deep portion passes into
the infraspinous fossa to anastomose with the suprascapular
artery. The dorsal scapular artery passes between
the roots of the brachial plexus and then deep
to the medial border of the scapula. The transverse
cervical artery arises from the thyrocervical trunk at
the root of the neck and can provide origin for a dorsal
scapular branch. The lateral thoracic and thoracoacromial
arteries are branches of the second part of the
axillary artery and provide no supply to the latissimus
dorsi.
GAS 683, 684, 697; GA 368

68
Q

A 31-year-old male hockey player fell on his elbow
and is admitted to the emergency department.
Radiographic examination reveals a fracture of the surgical
neck of the humerus, producing an elevation and
adduction of the distal fragment. Which of the following
muscles would most likely cause the adduction of
the distal fragment?
⃣ A. Brachialis
⃣ B. Teres minor
⃣ C. Pectoralis major
⃣ D. Supraspinatus
⃣ E. Pectoralis minor

A

C. The surgical neck of the humerus is a typical
site of fractures. The fracture line lies above the insertions
of the pectoralis major, teres major, and latissimus
dorsi muscles. The supraspinatus muscle abducts
the proximal fragment, whereas the distal fragment is
elevated and adducted. The elevation results from
contraction of the deltoid, biceps brachii, and coracobrachialis
muscles. The adduction is due to the action
of pectoralis major, teres major, and latissimus dorsi.
GAS 139-140, 659, 686-687; GA 374, 492

69
Q

A 74-year-old woman is admitted to the emergency
department after stumbling over her pet dog.
Radiographic examination reveals a fracture of the upper
third of the right radius, with the distal fragment of
the radius and hand pronated. The proximal end of the
fractured radius deviates laterally. Which of the following
muscles is primarily responsible for the lateral
deviation?
⃣ A. Pronator teres
⃣ B. Supinator
⃣ C. Pronator quadratus
⃣ D. Brachioradialis
⃣ E. Brachialis

A

B. The fracture line of the upper third of the radius
lies between the bony attachments of the supinator and
the pronator teres muscles. The distal radial fragment
and hand are pronated due to unopposed contraction of
pronator teres and pronator quadratus muscles. The
proximal fragment deviates laterally by the unopposed
contraction of the supinator muscle. The brachioradialis
inserts distally on the radius. The brachialis inserts on
the coronoid process of the ulna and would not be involved
in the lateral deviation of the radius.
GAS 736, 747-749; GA 390

70
Q

A 12-year-old male had received a laceration in
the palmar surface of the wrist while playing with a
very sharp knife. The cut ends of a tendon could be
seen within the wound in the exact midline of the
wrist. Which tendon lies in this position in most
people?
⃣ A. Palmaris longus
⃣ B. Flexor carpi radialis
⃣ C. Abductor pollicis longus
⃣ D. Flexor carpi ulnaris
⃣ E. Flexor pollicis longus

A

A. The palmaris longus passes along the midline
of the fl exor surface of the forearm. The fl exor carpi
radialis is seen in the lateral portion of the forearm
superfi cially, passing over the trapezium to insert at
the base of the second metacarpal. The abductor pollicis
longus tendon is laterally located in the wrist,
where it helps form the lateral border of the anatomic
snuffbox. The fl exor carpi ulnaris tendon can be seen
and palpated on the medial side of the wrist ventrally.
The fl exor pollicis longus tendon passes deep through
the carpal tunnel.
GAS 737-739; GA 374, 390, 398

71
Q

A 22-year-old male medical student was seen in
the emergency department with a complaint of pain in
his hand. He confessed that he had hit a vending machine
in the hospital when he did not receive his soft
drink after inserting money twice. The medial side of
the dorsum of the hand was quite swollen, and one of
his knuckles could not be seen when he “made a fi st.”
The physician made a diagnosis of a “boxer’s fracture.”
What was the nature of the impatient student’s
injury?
⃣ A. Fracture of the styloid process of the ulna
⃣ B. Fracture of the neck of the fi fth metacarpal
⃣ C. Colles’ fracture of the radius
⃣ D. Smith’s fracture of the radius
⃣ E. Bennett’s fracture of the thumb

A

B. The student had broken the neck of the fi fth
metacarpal when hitting the machine with his fi st.
This is the more common type of “boxer’s fracture.”
Neither a fracture of the ulnar styloid nor a Colles’
fracture nor a Smith fracture of the distal radius
would present with the absence of a knuckle as observed
here. Bennett’s fracture involves dislocation
of the carpometacarpal joint of the thumb. Indications
are that the injury is on the medial side of the
hand, not the wrist, nor the lateral side of the hand
or wrist.
GAS 769; GA 392, 394

72
Q

Fine motor function in the right hand of a 14-
year-old female with scoliosis since birth appeared to
be quite reduced, including thumb opposition, abduction
and adduction of the digits, and interphalangeal
joint extension. Radiography confi rmed that her severe
scoliosis was causing marked elevation of the right fi rst
rib. Long fl exor muscles of the hand and long extensors
of the wrist appear to be functioning within normal
limits. There is notable anesthesia of the skin on the
medial side of the forearm; otherwise, sensory function
in the limb is intact. Which of the following neural
structures is most likely impaired?
⃣ A. Median nerve
⃣ B. Middle trunk of the brachial plexus
⃣ C. Radial nerve
⃣ D. Lower trunk of the brachial plexus
⃣ E. T1 nerve root

A

E. Scoliosis (severe, lateral curvature of the
spine) in the patient is causing compression or
stretching of the T1 spinal nerve root by the fi rst rib
as the nerve ascends to join C8 and form the lower
trunk of the brachial plexus. T1 provides sensation for
the medial side of the forearm, via the medial antebrachial
cutaneous nerve from the medial cord of the
brachial plexus. T1 is the principal source of motor
supply to all of the intrinsic muscles in the palm. Its
dysfunction affects all fi ne motor movements of the
digits. Long fl exors of the fi ngers are intact; therefore,
the median nerve and ulnar nerve are not at fault.
The extensors of the wrist are functional; therefore,
the radial nerve is not paralyzed. The only sensory
disturbance is that of the T1 dermatome.
GAS 700-709; GA 361, 369-371

73
Q

A 23-year-old female had a painful injury to her
hand in a dry ski-slope competition, in which she fell
and caught her thumb in the matting. Radiographic
and physical examinations reveal rupture of the ulnar
collateral ligament of the metacarpophalangeal joint of
the thumb. Lidocaine is injected into the area to relieve
the pain, and she is scheduled for a surgical repair.
From which of the following clinical problems is she
suffering?
⃣ A. De Quervain’s syndrome
⃣ B. Navicular bone fracture
⃣ C. Boxer’s thumb
⃣ D. Gamekeeper’s thumb
⃣ E. Bennett’s thumb

A

D. Interestingly, “gamekeeper’s thumb” was a
term coined because this injury was most commonly
associated with Scottish gamekeepers who, it
is said, killed small animals such as rabbits by
breaking their necks between the ground and the
gamekeeper’s thumb and index fi nger. The resulting
valgus force on the abducted MCP joint caused injury
to the ulnar collateral ligament. These days this
injury is more commonly seen in skiers who land
awkwardly with their hand braced on a ski pole,
causing the valgus force on the thumb as is seen in
this patient. Whereas the term “skier thumb” is
sometimes used, “gamekeeper’s thumb” is still in
common usage.
GAS 755; GA 395

74
Q

A 26-year-old male power lifter visits the outpatient
clinic with a painful shoulder. Radiographic examination
reveals tendinopathy of the long head of the
biceps. Which of the following conditions will most
likely be present during physical examination?
⃣ A. Pain is felt in the anterior shoulder during
forced contraction.
⃣ B. Pain is felt in the lateral shoulder during
forced contraction.
⃣ C. Pain is felt during abduction and fl exion of
the shoulder joint.
⃣ D. Pain is felt during extension and adduction of
the shoulder joint.
⃣ E. Pain is felt in the lateral shoulder during fl exion
of the shoulder joint.

A

A. The long head of the biceps assists in shoulder
fl exion and during a tendinopathy would cause
pain in the anterior compartment of the shoulder,
where it originates at the supraglenoid tubercle. Also,
forced contraction would cause more of a greater tension
force on the tendon.
GAS 694, 715-716; GA 370

75
Q

A 43-year-old female tennis player visits the outpatient
clinic with pain over the right lateral epicondyle
of her elbow. Physical examination reveals that the
patient has lateral epicondylitis. Which of the following
tests should be performed during physical examination
to confi rm the diagnosis?
⃣ A. Nerve conduction studies
⃣ B. Evaluation of pain experienced during fl exion
and extension of the elbow joint
⃣ C. Observing the presence of pain when the
wrist is extended against resistance
⃣ D. Observing the presence of numbness and tingling
in the ring and little fi ngers when the
wrist is fl exed against resistance
E. ⃣ Evaluation of pain felt over the styloid process
of radius during brachioradialis contraction

A

C. The common extensor tendon originates from
the lateral epicondyle, and infl ammation of this tendon
is lateral epicondylitis, nicknamed “tennis elbow”
because the tendon is often irritated during the backhand
stroke in tennis. Because the extensors of the
wrist originate as part of the common extensor tendon,
extension of the wrist will exacerbate the pain of
lateral epicondylitis.
GAS 691; GA 380

76
Q

A male skier had a painful fall against a rocky
ledge. Radiographic fi ndings revealed a hairline fracture
of the surgical neck of the humerus. The thirdyear
medical student assigned to this patient was
asked to determine whether there was injury to the
nerve associated with the area of injury. Which of the
following tests would be best for checking the status
of the nerve?
⃣ A. Have the patient abduct the limb while holding
a 10-lb weight.
⃣ B. Have the patient shrug the shoulders.
⃣ C. Test for presence of skin sensation over the
lateral side of the shoulder.
⃣ D. Test for normal sensation over the medial
skin of the axilla.
⃣ E. Have the patient push against an immovable
object like a wall and assess the position of
the scapula.

A

C. The axillary nerve passes dorsally around
the surgical neck of the humerus (accompanied by
the posterior humeral circumfl ex artery) and can be
injured when the humerus is fractured at that location.
The axillary nerve provides sensation to the
skin over the upper, lateral aspect of the shoulder.
Therefore, although the patient might not be able to
abduct the arm because of the injury, a simple test
of skin sensation can indicate whether there is associated
nerve injury of the axillary nerve. Shrugging
the shoulders can help assess trapezius function,
thereby testing the spinal accessory nerve. Intact
sensation of the skin on the medial aspect of the
axilla or arm is an indication that the radial or intercostobrachial
nerves are functional. Pushing against
an immovable object tests the serratus anterior
muscle and the long thoracic nerve.
GAS 667; GA 355

77
Q

A 27-year-old male had lost much of the soft tissue
on the dorsum of his left hand in a motorcycle
crash. Imaging studies show no other upper limb injuries.
Because the left extensor carpi radialis longus and
brevis tendons were lost, it was decided to replace
those tendons with the palmaris longus tendons from
both forearms because of those tendons’ convenient
location and relative unimportance. Postoperatively it
is found that sensation is absent in both hands on the
lateral palm and palmar surfaces of the fi rst three digits;
there is also paralysis of thumb opposition. What is
the most likely cause of the sensory defi cit and motor
loss in both thumbs?
⃣ A. Bilateral loss of spinal nerve T1 with fractures
of fi rst rib bilaterally
⃣ B. Lower plexus (lower trunk) trauma
⃣ C. Dupuytren contracture
⃣ D. Left radial nerve injury in the posterior compartment
of the forearm
⃣ E. The palmaris longus was absent bilaterally;
the nerve beneath it looked like a tendon.

A

E. The surgeon took the distal segments of the
median nerves from both forearms, mistakenly believing
them to be palmaris longus tendons. Both of the
structures lie in the midline of the ventral surface of
the distal forearm and are often of similar appearance
in color and diameter. The nerve is located deep to
the tendon, when the tendon is present, but when the
tendon is absent, the nerve appears to be where the
tendon belongs. There is no evidence of rib fractures;
even so, a fractured rib would not explain loss of sensation
on the lateral portion of the palm. Lower
plexus trauma (C8, T1) would result in paralysis of
forearm fl exor muscles and all intrinsic hand muscles
and sensory loss over the medial dorsum of the hand,
in addition to palmar sensory loss. Dupuytren’s contracture
is a fl exion contracture of (usually) digits 4
and 5 from connective tissue disease in the palm.
Radial nerve injury in the posterior forearm would
affect metacarpophalangeal joint extension, thumb
extension, etc., not palmar disturbances.
GAS 743-744; GA 400

78
Q

A 15-year-old male received a shotgun wound to
the ventral surface of the upper limb. Upon examination
it is quickly observed that the patient exhibits a
complete clawhand but can extend his wrist. What is
the nature of this patient’s injury?
A. ⃣ The ulnar nerve has been severed at the wrist.
⃣ B. The median nerve has been injured in the
carpal tunnel.
⃣ C. The median and ulnar nerves are damaged at
the wrist.
⃣ D. The median and ulnar nerves have been injured
at the elbow region.
⃣ E. The median, ulnar, and radial nerves have
been injured at midhumerus.

A

C. Trauma both to the median and ulnar nerves
at the wrist results in total clawing of the fi ngers.
The metacarpophalangeal joints of all digits are extended
by the unopposed extensors because the radial
nerve is intact. All interossei and lumbricals are
paralyzed because the deep branch of the ulnar
nerve supplies all of the interossei; lumbricals I and
II are paralyzed, for they are innervated by the median
nerve; lumbricals III and IV are paralyzed, for
they receive supply from the deep ulnar nerve. The
interossei and lumbricals are responsible for extension
of the interphalangeal joints. When they are
paralyzed, the long fl exor tendons pull the fi ngers
into a position of fl exion, completing the “claw” appearance.
If the median nerve were intact, the clawing
would be less noticeable in the index and long
fi nger because the two lumbricals would still be capable
of some degree of extension of those interphalangeal
joints. If the median nerve alone is injured in
the carpal tunnel, there would be loss of thenar opposition
but not clawing. If the median and ulnar
nerves are both transected at the elbow, the hand
appears totally fl at because of the loss of long fl exors,
in addition to intrinsic paralysis.
GAS 726; GA 400

79
Q

A 68-year-old woman fell when she missed the
last step from her motor home. Radiographic examination
at the local medical care center reveals a fracture
of the distal radius. The distal fragment of the radius is
angled forward. What name is commonly applied to
this type of injury?
⃣ A. Colles’ fracture
⃣ B. Scaphoid fracture
⃣ C. Bennett’s fracture
⃣ D. Smith’s fracture
⃣ E. Boxer’s fracture

A

D. Colles’ fracture is a fracture of the distal radius
with the distal fragment displaced dorsally.
Smith’s fracture involves the distal fragment displaced
in a volar direction. Smith’s fracture is sometimes referred
to as a reverse Colles’ fracture.
GAS 756; GA 392

80
Q

It was reported by the sports media that the outstanding
27-year-old shortstop for the New York team
would miss a number of baseball games. He was hit on
a fi ngertip while attempting to catch a ball barehanded.
A tendon had been torn. The team doctor
commented that the ballplayer could not straighten the
last joint of the long fi nger of his right hand, and the
fi nger would require surgery. From what injury did the
ballplayer suffer?
⃣ A. Clawhand deformity
⃣ B. Boutonnière deformity
⃣ C. Swan-neck deformity
⃣ D. Dupuytren contracture
⃣ E. Mallet fi nger

A

E. The extensor tendons of the fi ngers insert distally
on the distal phalanx of each digit. If the tendon
is avulsed, or the proximal part of the distal phalanx
is detached, the distal interphalangeal joint is pulled
into total fl exion by the unopposed fl exor digitorum
profundus. This result gives the digit the appearance
of a mallet. In boutonnière deformity, the central portion
of the extensor tendon expansion is torn over the
PIP joint, allowing the tendon to move palmarward,
causing the tendon to act as a fl exor of the PIP joint.
This causes the DIP joint to be hyperextended. Swanneck
deformity involves slight fl exion of MCP joints,
hyperextension of PIP joints, and slight fl exion of DIP
joints. This condition results most often from shortening
of the tendons of intrinsic muscles, as in rheumatoid
arthritis. Dupuytren contracture results from connective
tissue disorder in the palm, usually causing
irreversible fl exion of digits 4 and 5. Clawhand occurs
with lesions to the median and ulnar nerves at the
wrist. In this clinical problem all intrinsic muscles are
paralyzed, including the extensors of the interphalangeal
joints. The MCP joint extensors, supplied by the
radial nerve, and the long fl exors of the fi ngers, supplied
more proximally in the forearm by the median
and ulnar nerves, are intact and are unopposed, pulling
the fi ngers into the “claw” appearance.
GAS 760; GA 392, 394, 422

81
Q

A 31-year-old female fi gure skater is examined in
the emergency department following an injury that
forced her to withdraw from competition. When her
male partner missed catching her properly from an
overhead position, he grasped her powerfully, but awkwardly,
by the forearm. Clinical examination demonstrated
a positive Ochsner test, inability to fl ex the
distal interphalangeal joint of the index fi nger on clasping
the hands. In addition, she is unable to fl ex the
terminal phalanx of the thumb and has loss of sensation
over the thenar half of the hand. What is the most
likely nature of her injury?
⃣ A. Median nerve injured within the cubital fossa
⃣ B. Anterior interosseous nerve injury at the pronator
teres
⃣ C. Radial nerve injury at its entrance into the
posterior forearm compartment
⃣ D. Median nerve injury at the proximal skin
crease of the wrist
E. ⃣ Ulnar nerve trauma halfway along the forearm

A

A. Because the median nerve is injured within
the cubital fossa, the long fl exors are paralyzed, including
the fl exor pollicis longus. Flexor pollicis longus
would not be paralyzed if the median nerve were
injured at the wrist. Lateral palm sensory loss con-
fi rms median nerve injury. If only the anterior interosseous
nerve were damaged, there would be no cutaneous
sensory defi cit. The radial nerve supplies wrist
extensors, long thumb abductor, and metacarpophalangeal
joint extensors. The ulnar nerve does not supply
sensation to the lateral palm.
GAS 651, 729, 731; GA 400

82
Q

A 19-year-old fell from a cliff when he was hiking
in the mountains. He broke his fall by grasping a
tree branch, but he suffered injury to the C8-T1 spinal
nerve roots. Sensory tests would thereafter con-
fi rm the nature of his neurologic injury by the sensory
loss in the part of the limb supplied by which of
the following?
⃣ A. Lower lateral brachial cutaneous nerve
⃣ B. Musculocutaneous nerve
⃣ C. Intercostobrachial nerve
⃣ D. Medial antebrachial cutaneous nerve
⃣ E. Median nerve

A

D. In a lesion of the lower trunk, or the C8 and
T1 nerve roots, there is sensory loss on the medial
forearm and the medial side of hand (dorsal and ventral).
The medial cord is the extension of the lower
trunk. The medial cord gives origin to the medial antebrachial
cutaneous nerve, which supplies the T1
dermatome of the medial side of the antebrachium.
The lower lateral brachial cutaneous nerve arises
from the radial nerve, C5 and C6. The musculocutaneous
nerve arises from the lateral cord, ending in the
lateral antebrachial cutaneous nerve, with C5 and C6
dermatome fi bers. The intercostobrachial nerve is the
lateral cutaneous branch of the T2 ventral primary
ramus and supplies skin on the medial side of the
arm. The median nerve distributes C6 and C7 sensory
fi bers to the lateral part of the palm, thumb, index,
long fi nger, and half of the ring fi nger.
GAS 743-745, 771-772; GA 384

83
Q

The mastectomy procedure on a 52-year-old female
involved excision of the tumor and a removal of
lymph nodes, including the pectoral, central axillary,
and infraclavicular groups. Six months after her mastectomy,
the patient complains to her personal physician
of an unsightly deep hollow area inferior to the
medial half of the clavicle, indicating a signifi cant area
of muscle atrophy and loss. She states that the disfi gurement
has taken place quite gradually since her mastectomy.
Physical examination reveals no obvious motor
or sensory defi cits. What was the most likely cause
of the patient’s cosmetic problem?
⃣ A. Part of the pectoralis major muscle was cut
and removed in the mastectomy.
⃣ B. The pectoralis minor muscle was removed entirely
in the surgery.
C. ⃣ A branch of the lateral pectoral nerve was cut.
⃣ D. The medial pectoral nerve was cut.
⃣ E. The lateral cord of the brachial plexus was
injured.

A

C. The fi rst branch of the lateral pectoral nerve is
typically the only source of motor supply to the clavicular
head of the pectoralis major. If it is injured (as in this
case of an iatrogenic injury when the infraclavicular
nodes were removed), this part of the muscle undergoes
atrophy, leaving an infraclavicular cosmetic defi cit.
The remainder of the lateral pectoral nerve joins the
medial pectoral nerve in a neural arch that provides
motor supply to the remaining parts of the pectoralis
major and the pectoralis minor. Physical examination
reveals no obvious motor or sensory defi cits. Loss of
the medial pectoral nerve would have no effect on the
clavicular head of pectoralis major and might not be
discernible. Injury to the lateral cord would lead to loss
not only of all of the lateral pectoral nerve but also the
musculocutaneous nerve, resulting in biceps and brachialis
paralysis and lateral antebrachial sensory loss.
GAS 688, 702, 706; GA 370

84
Q

A 54-year-old female was found unconscious on
the fl oor, apparently after a fall. She was admitted to
the hospital, and during physical examination it was
observed that she had absence of her brachioradialis
refl ex. Which spinal nerve is primarily responsible for
this refl ex in the majority of cases?
⃣ A. C5
⃣ B. C6
⃣ C. C7
⃣ D. C8
⃣ E. T1

A

B. The C6 spinal nerve is primarily responsible
for the brachioradialis refl ex. C5 and C6 are both involved
in the biceps brachii refl ex; C5 for motor, C6
for the sensory part of the refl ex arc; C7 is the key
spinal nerve in the triceps refl ex.
GAS 700-709; GA 361, 369-371

85
Q

A 43-year-old man is admitted to the hospital,
having suffered a whiplash injury when his compact
auto was struck from behind by a sports utility vehicle.
MRI examination reveals some herniation of a disk in
the cervical region. Physical examination reveals that
the patient has lost elbow extension; there is absence
of his triceps refl ex and loss of extension of the metacarpophalangeal
joints on the ipsilateral side. Which of
the following spinal nerves is most likely affected?
⃣ A. C5
⃣ B. C6
⃣ C. C7
⃣ D. C8
⃣ E. T1

A

C. C7 is the main spinal nerve that contributes
to the radial nerve and innervates the triceps. Absence
of the triceps refl ex is usually indicative of a C7
radiculopathy or injury.
GAS 700-709; GA 361, 369-371

86
Q

A 29-year-old patient has a dislocated elbow in
which the ulna and medial part of the distal humerus
have become separated. What classifi cation of joint is
normally formed between these two bones?
⃣ A. Trochoid
⃣ B. Ginglymus
⃣ C. Enarthrodial
⃣ D. Synarthrosis
⃣ E. Sellar

A

B. Ginglymus joint is the technical term to describe
a hinge joint. It allows motion in one axis
(fl exion and extension in the case of the elbow) and
is therefore a uniaxial joint. The other types of joints
listed allow motion in more than one axis.
GAS 79-81; GA 372

87
Q

A 45-year-old woman motorcyclist, propelled
over the handlebars of her bike by an encounter with
a rut in the road, lands on the point of one shoulder.
The woman is taken by ambulance to the emergency
department. During physical examination the arm appears
swollen, pale, and cool. Any movement of the
arm causes severe pain. Radiographic examination
reveals a fracture and a large hematoma, leading to
diagnosis of Volkmann’s ischemic contracture. At
which of the following locations has the fracture most
likely occurred?
⃣ A. Surgical neck of humerus
⃣ B. Radial groove of humerus
⃣ C. Supracondylar line of humerus
⃣ D. Olecranon
⃣ E. Lateral epicondyle

A

C. A fracture of the humerus just proximal to
the epicondyles is called a supracondylar fracture.
This is the most common cause of a Volkmann ischemic
fracture. The sharp bony fragment often lacerates
the brachial (or other) artery, with bleeding into
the fl exor compartment. Diminution of arterial supply
to the compartment results in the ischemia.
Bleeding into the compartment causes greatly increased
pressure, fi rst blocking venous outfl ow from
the compartment, then reducing the arterial fl ow
into the compartment. The ischemic muscles then
undergo unrelieved contracture. A humeral fracture
is sometimes placed in a cast from shoulder to wrist,
often concealing the ischemia until major tissue loss
occurs. Cold, insensate digits and great pain are
warnings of this compartmental problem, demanding
that the cast be removed and the compartment
opened (“released”) for pressure reduction and vascular
repair. Fracture of the surgical neck endangers
the axillary nerve and posterior humeral circumfl ex
artery, although not ischemic contracture. Fracture
of the humerus in the spiral groove can injure the
radial nerve and profunda brachii artery. Fracture of
the olecranon does not result in Volkmann’s contracture,
although the triceps brachii can displace the
distal fractured fragment of the ulna.
GAS 728; GA 355

88
Q

A 55-year-old female choreographer had been
treated in the emergency department after she fell from
the stage into the orchestra pit. Radiographs revealed
fracture of the styloid process of the ulna. Disruption of
the triangular fi brocartilage complex is suspected. With
which of the following bones does the ulna normally
articulate at the wrist?
⃣ A. Triquetrum
⃣ B. Hamate
⃣ C. Radius and lunate
⃣ D. Radius
⃣ E. Pisiform and triquetrum

A

D. Normally, the distal part of the ulna articulates
only with the radius at the distal radioulnar joint
at the wrist, a joint that participates in pronation/
supination. The head of the ulna does not articulate
with any of the carpal bones; instead, it is separated
from the triquetrum and lunate bones by the triangular
fi brocartilage complex between it and the radius.
The pisiform articulates with the triquetrum. The carpal
articulation of the radius is primarily that of the
scaphoid (old name is navicular) bone.
GAS 713, 714, 731-733; GA 391

89
Q

A 67-year-old female had a bad fall while walking
her dog the evening before. She states that she fell on
her outstretched hand. Radiographs do not demonstrate
any bony fractures. The clinician observes the
following signs of neurologic injury: There is weakness
of fl exion of her wrist in a medial direction, there is a
loss of sensation on the medial side of the hand, and
there is clawing of the fi ngers. Where is the most likely
place of nerve trauma?
⃣ A. Behind the medial epicondyle
⃣ B. Between the pisiform bone and the fl exor retinaculum
⃣ C. Within the carpal tunnel
⃣ D. At the cubital fossa, between the ulnar and
radial heads of origin of fl exor digitorum superfi
cialis
⃣ E. At the radial neck, 1 cm distal to the humerocapitellar
joint

A

A. The force of the woman’s fall on the outstretched
hand was transmitted up through the forearm,
sometimes resulting in dislocation of the olecranon
at the elbow, putting traction on the ulnar nerve
as it passes around the medial epicondyle of the humerus.
Ulnar trauma at the elbow can cause weakness
in medial fl exion (adduction) at the wrist, from
loss of the fl exor carpi ulnaris. Ulnar nerve injury also
results in sensory loss in the medial hand and paralysis
of the interossei and medial two lumbricals, with
clawing especially of digits 4 and 5. Injury of the ulnar
nerve at the pisiform bone would not affect the
fl exor carpi ulnaris, nor would it produce sensory loss
on the dorsum of the hand because the dorsal cutaneous
branch of the ulnar branches off proximal to the
wrist. Carpal tunnel problems affect median nerve
function, which is not indicated here. The ulnar nerve
passes medial to the cubital fossa between the heads
of the fl exor carpi ulnaris, not between the heads of
the fl exor digitorum superfi cialis. Injuries at the radial
neck affect the site of division of the radial nerve, and
its paralysis would not result in the clinical problems
seen in this patient.
GAS 727-728; GA 386

90
Q

An 18-year-old male suffered a signifi cant laceration
through the skin and underlying tissues at the
distal crease of the wrist. The medical student rotating
through the emergency department suspected (correctly)
that the ulnar nerve was cut completely through
at this location. Which of the following would most
likely occur?
⃣ A. The patient could not touch the tip of the
thumb to the tips of the other digits.
⃣ B. There would be loss of sensation on the dorsum
of the medial side of the hand.
⃣ C. The patient would be unable to fl ex the interphalangeal
joints.
⃣ D. There would be decreased ability to extend
the interphalangeal joints.
⃣ E. There would be no serious functional problem
at all to the patient.

A

D. The interossei are the most important muscles
in extension of the interphalangeal (IP) joints
because of the manner of their insertion into the extensor
expansion of the fi ngers, which passes dorsal
to the transverse axes of these joints. The lumbrical
muscles assist in IP extension, in addition to fl exing
the metacarpophalangeal joints. Ulnar nerve injury at
the wrist results in paralysis of all the interossei and
the medial two lumbricals. Extensors of the MCP
joints are innervated by the deep radial nerve. Unopposed
extension of the MCP joints causes them to be
held in extension whereas unopposed long fl exors of
the fi ngers (supplied by median and ulnar nerves
proximally in the forearm) cause them to be fl exed
into the “claw” position. The lumbricals of digits 2
and 3 are still intact because they are supplied by the
median nerve, so clawing is not seen as much on
these digits. Loss of opposition would result from
median or recurrent nerve paralysis. If the ulnar nerve
is cut at the wrist, its dorsal cutaneous branch to the
dorsum of the hand is unaffected.
GAS 655, 755; GA 395

91
Q

A 45-year-old man visits the outpatient clinic after
a digit of his left hand was injured when a door was
slammed on his hand. A superfi cial cut on his middle
fi nger has been sutured, but functional defi cits are observed
in the fi nger: The proximal interphalangeal joint
is pulled into constant fl exion, whereas the distal interphalangeal
joint is held in a position of hyperextension.
What is the most likely diagnosis?
⃣ A. Mallet fi nger
⃣ B. Boutonnière deformity
⃣ C. Dupuytren contracture
⃣ D. Swan-neck deformity
⃣ E. Silver fork wrist deformity

A

B. In boutonnière deformity, the central portion
of the extensor tendon expansion is torn over the PIP
joint, allowing the tendon to move palmarward, causing
the tendon to act as a fl exor of the PIP joint. This
causes the DIP joint to be hyperextended. The tear in
the extensor tendon is said to resemble a buttonhole
(“boutonnière” in French), and the head of the proximal
phalanx may stick through the hole.
GAS 732-755; GA 395

92
Q

A 67-year-old housepainter visits the outpatient
clinic complaining that his hands are getting progressively
worse, becoming more and more painful and
losing their function. On physical examination of the
hands, there is fl exion of the metacarpophalangeal
joints, extension of the proximal interphalangeal joints,
and slight fl exion of the distal interphalangeal joints.
What is the most likely diagnosis?
⃣ A. Mallet fi nger
⃣ B. Boutonnière deformity
⃣ C. Dupuytren contracture
⃣ D. Swan-neck deformity
⃣ E. Silver fork wrist deformity

A

D. Swan-neck deformity involves slight fl exion
of MCP joints, hyperextension of PIP joints, and slight
fl exion of DIP joints. This condition results most often
from shortening of the tendons of intrinsic muscles,
as in rheumatoid arthritis. When asked to straighten
the injured fi nger, the patient is unable to do so and
the curvature of the fi nger somewhat resembles the
neck of a swan.
GAS 732-755; GA 395

93
Q

Several weeks after surgical dissection of her left
axilla for the removal of lymph nodes for staging and
treatment of her breast cancer, a 32-year-old woman
was told by her general physician that she had “winging”
of her left scapula when she pushed against resistance
during her physical examination. She told the
physician that she had also experienced diffi culty lately
in raising her right arm above her head when she was
combing her hair. In a subsequent consult visit with
her surgeon, she was told that a nerve was accidentally
injured during the diagnostic surgical procedure and
that this produced her scapular abnormality and inability
to raise her arm normally. What was the origin of
this nerve?
⃣ A. The upper trunk of her brachial plexus
⃣ B. The posterior division of the middle trunk
⃣ C. Roots of the brachial plexus
⃣ D. The posterior cord of the brachial plexus
⃣ E. The lateral cord of the brachial plexus

A

C. The long thoracic nerve was injured during the
axillary dissection, resulting in paralysis of the serratus
anterior. The serratus anterior is important in rotation
of the scapula in raising the arm above the level of the
shoulder. Its loss results in protrusion of the inferior
angle (“winging” of the scapula), which is more obvious
when one pushes against resistance. The long
thoracic nerve arises from brachial plexus roots C5, C6,
and C7. The upper trunk (C5, C6) supplies rotator and
abductor muscles of the shoulder and elbow fl exors.
The posterior division of the middle trunk contains C7
fi bers for distribution to extensor muscles; likewise, the
posterior cord supplies extensors of the arm, forearm,
and hand. The lateral cord (C5, C6, and C7) gives origin
to the lateral pectoral nerve, the musculocutaneous
nerve, and the lateral root of the median nerve. There
is no sensory loss in the limb in this patient; injury to
any of the other nerve elements listed here would be
associated with specifi c dermatome losses.
GAS 700-709; GA 361, 369-371

94
Q

A 72-year-old man consulted his physician
because he had noticed a thickening of the skin at
the base of his left ring fi nger during the preceding
3 months. As he described it, “There appears to be
some hard tissue that is pulling my little and ring fi ngers
into my palm.” On examination of the palms of
both hands, localized and fi rm ridges are observed in
the palmar skin that extend from the middle part of the
palm to the base of the ring and little fi ngers. What is
the medical term for this sign?
⃣ A. Ape hand
⃣ B. Dupuytren
⃣ C. Clawhand
⃣ D. Wrist drop
⃣ E. Mallet fi nger

A

B. Dupuytren contracture or deformity is a result
of fi bromatosis of palmar fascia, resulting in irregular
thickening of the fascial attachments to the skin,
which causes gradual contraction of the digits, especially
digits 4 and 5. In 50% of cases, it is bilateral in
occurrence. Ape hand, or fl at hand, is a result of loss
of the median and ulnar nerves at the elbow, with
paralysis of all long fl exors of the fi ngers and all intrinsic
hand muscles. Clawhand results from paralysis of
interphalangeal joint extension by interossei and lumbricals,
innervated primarily by the ulnar nerve. Wrist
drop occurs with radial nerve paralysis and loss of the
extensors carpi radialis longus and brevis. Mallet fi nger
results from detachment of the extensor mechanism
from the distal phalanx of a fi nger and unopposed
fl exion of that distal interphalangeal joint.
GAS 758; GA 398

95
Q

A 24-year-old female basketball player is admitted
to the emergency department after an injury to her
shoulder. Radiographic examination reveals a shoulder
dislocation. What is the most commonly injured nerve
in shoulder dislocations?
⃣ A. Axillary
⃣ B. Radial
⃣ C. Median
⃣ D. Ulnar
⃣ E. Musculocutaneous

A

A. The axillary nerve is a direct branch of the
posterior cord and wraps around the surgical neck
of the humerus to innervate the teres minor and the
deltoid muscles. With this anatomic arrangement,
the axillary nerve is tightly “tethered” to the proximal
humerus. When the head of the humerus is
dislocated, it often puts traction on the axillary
nerve.
GAS 674; GA 631

96
Q

A 45-year-old male is admitted to the hospital
with a painful arm after a “strongest man in the world”
contest. Physical examination gives evidence of a rupture
of the long tendon of the biceps brachii ( Fig. 6-5 ).
Which of the following is the most likely location of the
rupture?
⃣ A. Intertubercular groove
⃣ B. Midportion of the biceps muscle
⃣ C. Junction with the short head of the biceps
muscle
⃣ D. Proximal end of the combined biceps muscle
⃣ E. Bony insertion of the muscle

A

A. The long head of the biceps brachii muscles
runs in the intertubercular groove on the proximal
humerus as it changes direction and turns medially to
attach to the supraglenoid tubercle of the scapula.
This change in direction within an osseous structure
predisposes the tendon to wear and tear, particularly
in people who overuse the biceps muscle. This type
of injury presents with a characteristic sign called the
“Popeye sign.”
GAS 694, 715-716, 735; GA 370

97
Q

After the orthopedic surgeon examined the MRI
of the shoulder of a 42-year-old female he informed her
that the supraspinatus muscle was injured and needed
to be repaired surgically. Which of the following is true
of the supraspinatus muscle?
⃣ A. It inserts on the lesser tubercle of the humerus.
⃣ B. It initiates adduction of the shoulder.
⃣ C. It is innervated chiefl y by the C5 spinal nerve.
⃣ D. It is supplied by the upper subscapular nerve.
⃣ E. It originates from the lateral border of the
scapula.

A

C. The supraspinatus muscle inserts on the
greater tubercle of the humerus and is said to initiate
abduction of the arm at the shoulder. It is supplied
principally by spinal nerve C5. The subscapularis
muscle is the only muscle that inserts on the
lesser tubercle. The subscapularis muscle is innervated
by the upper and lower subscapular nerves.
The teres minor takes origin from the lateral border
of the scapula; the teres major takes origin from the
region of the inferior angle and the lateral border of
the scapula.
GAS 678-679; GA 369

98
Q

A 5-year-old boy is admitted to the emergency
department after falling from a tree. The parents are
informed by the radiologist that their son’s fracture is
the most common fracture that occurs in children.
Which of the following bones was broken?
⃣ A. Humerus
⃣ B. Radius
⃣ C. Ulna
⃣ D. Scaphoid
⃣ E. Clavicle

A

E. During a fall on an outstretched upper limb,
the forces are conducted through the hand on up
through the bones of the limb in succession. Often
these bones do not fracture but rather pass the compressive
forces proximally. The appendicular skeleton
joins with the axial skeleton at the sternoclavicular
joint. The forces are not suffi ciently transferred
to the sternum, causing the clavicle to absorb
the force, resulting in fracture of this sigmoidalshaped
bone.
GAS 673; GA 4, 6-7, 56-57, 65, 108, 351

99
Q

A 22-year-old woman visits the outpatient clinic
with pain in her left upper limb. She has a long history
of pain in this limb and diffi culty with fi ne motor tasks
of the hand. Physical examination reveals paraesthesia
along the medial surface of the forearm and palm and
weakness and atrophy of gripping muscles (long fl exors)
and the intrinsic muscles of the hand. The radial
pulse is diminished when her neck is rotated to the
ipsilateral side (positive Adson test). What is the most
likely diagnosis?
⃣ A. Erb-Duchenne paralysis
⃣ B. Aneurysm of the brachiocephalic artery, with
plexus compression
⃣ C. Thoracic outlet syndrome
⃣ D. Carpal tunnel syndrome
⃣ E. Injury to the medial cord of the brachial
plexus

A

C. The patient is suffering from thoracic outlet
syndrome, involving neural and vascular elements.
This results from any condition that decreases the
dimensions of the superior thoracic aperture. It
could be a result of a cervical rib, accessory muscles,
and/or atypical connective tissue bands at the root
of the neck. In this case, symptoms involve the arm,
forearm, and hand. Paraesthesia along the medial
forearm and hand and atrophy of long fl exors and
intrinsic muscles point to a possible compression or
traction problem of the lower trunk (C8, T1) rather
than a lesion of either the median or ulnar nerve.
The lateral palm has no sensory problem, which
tends to rule out median nerve involvement. Changes
in the radial pulse point to vascular compression.
Erb-Duchenne paralysis of the upper trunk would
affect proximal limb functions, such as arm rotation,
abduction, etc. This lesion is on the left side, so the
brachiocephalic artery could not be involved because
it arises from the right side of the aortic arch;
moreover, it would not compress the brachial plexus.
Carpal tunnel syndrome would not explain the problems
of the forearm and medial hand, or the long
fl exor atrophy. An isolated medial cord lesion would
not explain the atrophy of all long fl exors and intrinsic
muscles and does not explain the radial pulse
characteristics. The ischemic pain in the arm is due
to vascular compression.
GAS 147; GA 370

100
Q

Physical examination reveals weakness of medial
deviation of the wrist (adduction), loss of sensation
on the medial side of the hand, and clawing
of the fi ngers. Where is the most likely place of
injury?
⃣ A. Compression of a nerve passing between the
humeral and ulnar heads of origin of fl exor
carpi ulnaris
⃣ B. Compression of a nerve passing at Guyon’s
canal between the pisiform bone and fl exor
retinaculum
⃣ C. Compression of a nerve passing through the
carpal tunnel
⃣ D. Compression of a nerve passing between the
ulnar and radial heads of origin of fl exor digitorum
superfi cialis
⃣ E. Compression of a nerve passing deep to brachioradialis
muscle

A

A. The ulnar nerve enters the forearm by
passing between the two heads of the fl exor carpi
ulnaris and descends between and innervates the
fl exor carpi ulnaris (for medial wrist deviation) and
fl exor digitorum profundus (medial half) muscles.
Injuring the ulnar nerve results in clawhand. It enters
the hand superfi cial to the fl exor retinaculum
and lateral to the pisiform bone, where it is vulnerable
to damage. The ulnar nerve also enters Guyon’s
canal, but damage to it here would not present with
the aforementioned symptoms. The median nerve
enters the carpal tunnel and the radial nerve passes
deep to the brachioradialis.
GAS 737; GA 398

101
Q

A 22-year-old pregnant woman was admitted
emergently to the hospital after the baby had begun to
appear at the introitus. The baby had presented in the
breech position, and it had been necessary to exert
considerable traction to complete the delivery. The
newborn is shown in Fig. 6-6 . Which of the following
structures was most likely injured by the trauma of
childbirth?
⃣ A. Radial nerve
⃣ B. Upper trunk of the brachial plexus
⃣ C. Lower trunk of the brachial plexus
⃣ D. Median, ulnar, and radial nerves
⃣ E. Upper and lower trunks of the brachial
plexus

A

B. During a breech delivery as described here,
downward traction is applied to the shoulders and
upper limbs as the baby is forcibly extracted from the
birth canal. This exerts traction on the upper cord of
the brachial plexus, often causing a traction injury
from which the baby can often recover. If the roots of
C5 and C6 are avulsed from the spinal cord, the injury
is permanent.
GAS 700-709; GA 361, 369-371

102
Q

A 17-year-old female student of martial arts entered
the emergency department with a complaint of
pain in her hand. Patient history reveals that she had
been breaking concrete blocks with her hand. Examination
reveals that the patient has weak abduction and
adduction of her fi ngers but has no diffi culty in fl exing
them. The patient also has decreased sensation over the
palmar surfaces of the fourth and fi fth digits. Which of
the following best describes the nature of her injury?
⃣ A. Compression of the median nerve in the carpal
tunnel
⃣ B. Fracture of the triquetrum, with injury to the
dorsal ulnar nerve
⃣ C. Dislocation of a bone in the proximal row of
the carpus
⃣ D. Fracture of the shaft of the fi fth metacarpal
⃣ E. Injury of the ulnar nerve in Guyon’s canal

A

E. Striking the concrete blocks with the medial
side of her hand has injured the ulnar nerve in Guyon’s
canal. This is the triangular tunnel formed by the pisiform
bone medially, the fl exor retinaculum dorsally,
and the deep fascia of the wrist ventrally. This injury
would result in loss of sensation to the medial palm
and the palmar surface of the medial one and a half
digits and motor loss of the hypothenar muscles, the
interossei, and the medial two lumbricals. The median
nerve is not involved, for the thenar muscles and lateral
palmar sensations are intact. The dorsal ulnar
nerve arises proximal to the wrist, thus it would not be
lost. Carpal dislocation is unlikely. If the lunate bone
were dislocated, it would not cause compression of the
ulnar nerve at the wrist. There is no indication of fi fth
metacarpal fracture, the so-called boxer’s fracture.
GAS 744; GA 417

103
Q

A 10-year-old male suffered a dog bite that entered
the common fl exor synovial sheath of his forearm.
He was admitted to the hospital, where the
wound was cleaned and dressed and he was treated
further with rabies antiserum. Two days later the boy
was suffering from an elevated temperature, and his
palm and one digit were obviously swollen, causing
him to cry with pain. Into which of the digits could the
infection spread most easily, following the anatomy of
the typical common fl exor sheath?
⃣ A. First
⃣ B. Second
⃣ C. Third
⃣ D. Fourth
⃣ E. Fifth

A

E. The common fl exor sheath encloses the long
fl exor tendons of the fi ngers. This sheath is usually
continuous with the fl exor sheath of the little fi nger,
which continues within the palm, having no connection
with sheaths of the other digits, which do not
extend into the palm.
GAS 755; GA 410

104
Q

While sharpening his knife, a 23-year-old male
soldier accidentally punctured the ventral side of the
fi fth digit at the base of the distal phalanx. The wound
became infected, and within a few days the infection
has spread into the palm, within the sheath of the
fl exor digitorum profundus tendons. If the infection
were left untreated, into which of the following spaces
could it most likely spread?
⃣ A. Central compartment
⃣ B. Hypothenar compartment
⃣ C. Midpalmar space
⃣ D. Thenar compartment
⃣ E. Thenar space

A

C. The infectious agent was introduced into the
synovial sheath of the long tendons of the little (fi fth)
fi nger. Proximally, this sheath runs through the midpalmar
space, and infl ammatory processes typically
rupture into this space unless aggressively treated
with the appropriate antibiotics.
GAS 768; GA 410

105
Q

A 36-year-old patient is admitted to the emergency
department with a dull ache in the shoulder or
axilla ( Fig. 6-7 ). During physical examination the pain
worsens by activity, and, conversely, rest and elevation
relieve the pain. History reveals that the patient was
hospitalized the past week and a central venous line
was used. What is the most likely diagnosis?
⃣ A. Axillary-subclavian vein thrombosis
⃣ B. Compression of C5 to C8 spinal nerve
⃣ C. Disk herniation of C4 to C8
⃣ D. Impingement syndrome
⃣ E. Injury to radial, ulnar, and median nerves

A

A. Axillary-subclavian vein thrombosis is becoming
much more common in recent years because of
the extensive use of catheters in cancer patients and
other chronic medical conditions. Effort-induced thrombosis
is seen with strenuous use of the dominant arm
with hyperabduction and external rotation of the arm or
backward and downward rotation of the shoulder as in
playing cricket, volleyball, or baseball or chopping
wood. Because the symptoms of subclavian stenosis are
fairly dramatic, most patients present promptly, usually
within 24 hours. They complain of a dull ache in the
shoulder or axilla, the pain worsened by activity. Conversely,
rest and elevation often relieve the pain. Patients
with catheter-associated axillary-subclavian deep
vein thrombosis report similar symptoms at the arm or
shoulder on the side with the indwelling catheter.
GAS 722; GA 377

106
Q

A 22-year-old woman had suffered a severe knife
wound to the upper lateral portion of her pectoral region,
with entry of the knife at the deltopectoral groove.
Pressure applied to the wound had prevented further
profuse bleeding. In the emergency department, vascular
clamps were applied to the axillary artery, proximal
and distal to the site of injury—which had occurred
between the second and third parts of the axillary artery.
The vascular surgeon knew there was time to repair
the wound of the artery because of the rich collateral
pathway provided by the anastomoses between
which of the following of arteries?
⃣ A. Transverse cervical and suprascapular
⃣ B. Posterior humeral circumfl ex and profunda
brachii
⃣ C. Suprascapular and circumfl ex scapular
⃣ D. Supreme (superior) thoracic and
thoracoacromial
⃣ E. Lateral thoracic and suprascapular

A

C. The injury is at the second part of the axillary
artery. The suprascapular artery is a branch of the thy-
rocervical trunk off the subclavian artery, proximal to
the axillary artery. The subscapular artery is the major
branch of the third part of the axillary artery, giving of
the thorocodorsal and the circumfl ex scapular. In this
case blood would be fl owing from the circumfl ex
scapular artery in a retrograde direction into the axillary
artery, supplying blood distal to the injury.
GAS 695; GA 368

107
Q

In a penetrating wound to the forearm of a
24-year-old male, the median nerve is injured at the
entrance of the nerve into the forearm. Which of the
following would most likely be apparent when the patient’s
hand is relaxed?
⃣ A. The MCP and IP joints of the second and
third digits of the hand will be in a condition
of extension.
⃣ B. The third and fourth digits will be held in a
slightly fl exed position.
⃣ C. The thumb will be fl exed and slightly
abducted.
⃣ D. The fi rst, second, and third digits will be held
in a slightly fl exed position.
⃣ E. The MCP and IP joints of the second and
third digits of the hand will be in a condition
of fl exion.

A

A. This proximal injury to the median nerve
would paralyze all of the long fl exors of the digits, except
for the DIP fl exors of digits 4 and 5, thereby swinging
the “balance of power” to the muscles that extend
the digits, all of which are innervated by the radial
nerve. The intrinsic hand muscles can aid in fl exion of
the MCP joints, and they are innervated by the ulnar
nerve. However, they are of insuffi cient size to compensate
for the extensor forces exerted on fi ngers.
GAS 518, 611, 612; GA 395

108
Q

A 55-year-old male fi refi ghter is admitted to the
hospital after blunt trauma to his right axilla. Examination
reveals winging of the scapula and partial paralysis
of the right side of the diaphragm. Which of the following
parts of the brachial plexus have been injured?
⃣ A. Cords
⃣ B. Divisions
⃣ C. Roots
⃣ D. Terminal branches
⃣ E. Trunks

A

C. The winged scapula results from a lesion of
the long thoracic nerve, which supplies the serratus
anterior muscle. This muscle is responsible for rotating
the scapula upward, which occurs during abduction
of the arm above the horizontal. The long thoracic
nerve comes off the C5 to C7 roots of the
brachial plexus. The diaphragm is supplied by the
phrenic nerve, which comes off the spinal nerve roots
C3 to C5.
GAS 700-709; GA 361, 369-371

109
Q

A 69-year-old man has numbness and pain in the
middle three digits of his right hand at night. He retired
9 years ago after working as a carpenter for 30 years. He
has atrophy of the thenar eminence (see Fig. 6-2on
page 172). Which of the following conditions will be the
most likely cause of this atrophy?
⃣ A. Compression of the median nerve in the carpal
tunnel
⃣ B. Formation of the osteophytes that compress
the ulnar nerve at the ulnar condyle
⃣ C. Hypertrophy of the triceps muscle compressing
the brachial plexus
⃣ D. Osteoarthritis of the cervical spine
⃣ E. Repeated trauma to the ulnar nerve

A

A. The median nerve supplies sensory innervation
to the thumb, index, and middle fi nger as
well as to the lateral half of the ring fi nger. The median
nerve also provides motor innervation to muscles
of the thenar eminence. Compression of the
median nerve in the carpal tunnel explains these
defi cits in conjunction with normal functioning of
the fl exor compartment of the forearm. The ulnar
nerve is not implicated in these symptoms. Compression
of the brachial plexus could not be attributed to
pressure from hypertrophy of the triceps, it is located
distal to the plexus. In addition, symptoms would
include several upper limb defi cits rather than the
focal symptoms described in this instance. Osteoarthritis
of the cervical spine would also lead to increasing
complexity of symptoms.
GAS 764, 788; GA 406

110
Q

A 54-year-old woman presents with pain in her
right wrist that resulted when she fell forcefully on her
outstretched hand. Radiographic studies indicate an
anterior dislocation of a carpal bone of the proximal
row (see Fig. 6-3 on page 180). Which of the following
bones is most commonly dislocated?
⃣ A. Capitate
⃣ B. Lunate
⃣ C. Scaphoid
⃣ D. Pisiform
⃣ E. Triquetrum

A

B. The lunate bone is the most commonly dislocated
bone. Displacement is always anteriorly. Dislocation
of the lunate bone can precipitate the signs
associated typically with carpal tunnel syndrome.
GAS 752-754; GA 392, 394, 422

111
Q

A 32-year-old male who is an expert target
shooter reports pain in his right upper limb and slight
tingling and numbness of all digits of the ipsilateral
hand. However, the tingling and numbness of the
fourth and fi fth digits is the most severe. The man
states that the problem usually occurs when he is fi ring
his gun with his hand overhead. Radiographic studies
reveal the presence of a cervical rib and accessory scalene
musculature. Which of the following structures is
most likely being compressed?
⃣ A. Axillary artery
⃣ B. Upper trunk of brachial plexus
⃣ C. Subclavian artery
⃣ D. Lower trunk of brachial plexus
⃣ E. Brachiocephalic artery and lower trunk of brachial
plexus

A

D. A cervical rib (found at C7) typically causes
thoracic outlet syndrome, which is a condition characterized
by weak muscle tone in the hand and loss
of radial pulse when the upper limb is abducted
above the shoulder. The mechanism of injury with the
gun being fi red overhead suggests a lower trunk injury
to the brachial plexus. The axillary artery supplies
the shoulder muscles, and there is no loss of
function to these muscles. The upper trunk of the
brachial plexus also supplies innervation to the shoulder
muscles, which are unaffected based on the patient’s
presenting abnormalities. The subclavian artery
is located anterior to the brachial plexus until it
separates the cords as it passes under the clavicle.
The brachiocephalic artery and lower trunk of the
brachial plexus is only partially correct; the brachiocephalic
artery is not directly associated with the
brachial plexus due to its location at the midline of
the body behind the sternum.
GAS 700-709; GA 61, 369-371

112
Q

A 23-year-old woman arrives at the emergency
department with a swollen, painful forearm. An MRI
examination reveals a compartment syndrome originating
at the interosseous membrane between the radius
and ulna. Which of the following type of joint will most
likely be affected?
⃣ A. Synarthrosis
⃣ B. Symphysis
⃣ C. Synchondrosis
⃣ D. Trochoid
⃣ E. Ginglymus

A

A. A synarthrosis joint is a fi brous connection
that allows minimal to no movement. In this case,
virtually no movement is allowed by the interosseous
membrane joint between the radius and ulna. Symphysis
joints are permanent fi brocartilaginous fusions
between two bones; pubic symphysis is an example.
Synchondrosis is a temporary joint made of cartilage
that transitions to bone typically after growth completes
(i.e., epiphyseal plate). Trochoid joints are
pivot joints, and the humeral-radial portion of the elbow
joint is an example. Ginglymus joints are hinge
joints located at the interphalangeal junctions in the
hand and foot (PIPs and DIPs).
GAS 731, 734-735; GA 396

113
Q

While working out with weights, a 28-year-old
woman experiences a severe pain in her chest. The
pain is referred to the anterior chest wall and radiating
to the mandible and her left arm. The woman felt dizzy
and after 10 minutes she collapsed and was unconscious.
A physician happened to be near the woman
and immediately tried to feel her radial pulse. The radial
artery lies between two tendons near the wrist,
which are useful landmarks. Which of the following is
the correct pair of tendons?
⃣ A. Flexor carpi radialis and palmaris longus
⃣ B. Flexor carpi radialis and brachioradialis
⃣ C. Brachioradialis and fl exor pollicis longus
⃣ D. Flexor pollicis longus and fl exor digitorum superfi
cialis
⃣ E. Flexor pollicis longus and fl exor digitorum
profundus

A

B. The radial pulse is best located on the anterior
forearm (antebrachium) just proximal to the wrist
joint. At this point the radial artery travels on the
distal radius between the fl exor carpi radialis and
brachioradialis tendons. The palmaris longus tendon
travels more medially to the radial artery and above
the fl exor retinaculum. The fl exor pollicis longus tendon
is a deeper structure in the antebrachium and is
also located medially to the radial artery.
GAS 785; GA 374, 390

114
Q

A 59-year-old woman is admitted to the hospital
in a state of shock. During physical examination, several
lacerations are noted in her forearm and her radial
pulse is absent. Where is the most typical place to identify
the radial artery immediately after crossing the radiocarpal
joint?
⃣ A. Between the two heads of the fi rst dorsal interosseous
muscles
⃣ B. At the anatomic snuffbox
⃣ C. Below the tendon of the fl exor pollicis longus
⃣ D. Between the fi rst and second interosseous
muscle
⃣ E. Between the fi rst interosseous muscle and the
adductor pollicis longus

A

B. The radial artery enters the palm through
the anatomic snuffbox. The artery then moves on to
pierce through the two heads of the fi rst dorsal interosseous
muscle and enter the deep aspect of the
palm. The fl exor pollicis longus tendon runs on the
palmar aspect of the hand and the radial artery runs
on the dorsal aspect of the hand before entering the
UPPER LIMB
206
deep aspect of the palm, and therefore the radial artery
does not run below this tendon. The radial artery
does not run between the fi rst and second interosseous
muscle and therefore cannot be used as a landmark
to identify the artery. Finally, the artery does not
run between the fi rst interosseous muscle and the adductor
pollicis longus.
GAS 781; GA 422

115
Q

A 69-year-old woman visits the outpatient clinic
with a complaint of numbness and tingling of her hand
for the past 3 months. Physical examination reveals she
has numbness and pain in the lateral three digits of her
right hand that are relieved by vigorous shaking of the
wrist. In addition, the abductor pollicis brevis, opponens
pollicis, and the fi rst two lumbrical muscles are weakened.
Sensation was decreased over the lateral palm and
the volar aspect of the fi rst three digits. Which of the
following nerves is most likely compressed?
⃣ A. Ulnar
⃣ B. Radial
⃣ C. Recurrent median
⃣ D. Median
⃣ E. Posterior interosseous

A

D. The median nerve provides innervation to
the fl exor compartment of the forearm; cutaneous innervation
of the second, third, and fourth digits and
palmar and dorsum aspects of the hand; and innervation
of four intrinsic hand muscles: fi rst and second
lumbricals, abductor pollicis brevis, opponens pollicis,
and fl exor pollicis brevis. The thenar compartment
contains the muscle abductor pollicis brevis,
opponens pollicis, and fl exor pollicis brevis, and these
muscles are innervated by the recurrent branch of the
median nerve. The patient has weakening of the fi rst
two lumbricals and not simply the thenar muscles, so
the median nerve is most likely to be compressed.
Another indication that the median nerve is compressed
is the vigorous shaking of the wrist. Because
the median nerve traverses the carpal tunnel, carpal
tunnel compression could lead to this action on part
of the patient. The ulnar nerve provides innervation
for part of the fl exor digitorum profundus and fl exor
carpi ulnaris. These muscles are not weakened in this
patient. The radial nerve provides cutaneous supply
to the dorsum of the hand and forearm as well as
extensor muscles of the forearm. The posterior interosseous
nerve is a branch of the radial nerve and
provides innervation of the extensor muscles in the
forearm.
GAS 724-731; GA 361

116
Q

A 32-year-old man is admitted to the emergency
department after a severe car crash. Radiographic examination
reveals multiple fractures of his right upper
limb. A surgical procedure is performed and metallic
plates are attached to various bony fragments to restore
the anatomy. Five months postoperatively the
patient visits the outpatient clinic. Upon physical examination
the patient can abduct his arm and extend
the forearm, but the sensation of the forearm and
hand is intact; however, the hand grasp is very weak,
and he cannot extend his wrist against gravity. Which
of the following nerves was most likely injured during
the surgical procedure?
⃣ A. Posterior cord of the brachial plexus
⃣ B. Radial nerve at the distal third of the humerus
⃣ C. Radial and ulnar
⃣ D. Radial, ulnar, and median
⃣ E. Radial and musculocutaneous

A

B. Radial nerve at the distal third of the humerus.
The patient can extend his forearm, which suggests
that the triceps muscle is not weakened. Supination
appears to be weak along with hand grasp and
wrist drop. This would indicate that part of the radial
nerve has been lost below the innervation of the triceps
and above the branches to the supinator and extensors
in the forearm. However, sensation on the forearm and
hand is intact, indicating that the superfi cial branch of
the radial nerve is intact. The superfi cial branch of the
radial nerve divides from the deep radial nerve at
the distal third of the humerus. The posterior cord of
the brachial plexus is responsible for providing innervation
of the axially and radial nerves. This patient
does have some radial nerve innervation and no loss of
axillary nerve function. The patient does not have
weakened adduction of the wrist, indicating that the
ulnar nerve is not injured. If both the radial and musculocutaneous
nerves are injured, supination would
not be possible as the supinator muscle and biceps
provide supination of the forearm.
GAS 750; GA 376

117
Q

A 52-year-old man is admitted to the emergency
department after falling on wet pavement. Radiographic
examination reveals fracture of the radius. An
MRI study reveals a hematoma between the fractured
radius and supinator muscle. Upon physical examination
the patient has weakened abduction of the thumb
and extension of the metacarpophalangeal joints of the
fi ngers. Which of the following nerves is most likely
affected?
⃣ A. Anterior interosseous
⃣ B. Posterior interosseous
⃣ C. Radial nerve
⃣ D. Deep branch of ulnar nerve
⃣ E. Median nerve

A

B. The posterior interosseous nerve is an extension
of the deep branch of the radial nerve. It is
responsible for innervation of several muscles in the
extensor compartment of the posterior aspect of the
forearm, including extension of the metacarpophalangeal
joints. The posterior interosseous nerve
courses laterally around the radius and passes between
the two heads of the supinator muscle and is
thus likely to be compressed by a hematoma between
the fractured radius and the supinator muscle.
Though the radial nerve gives rise to the posterior
interosseous nerve, this answer choice is too vague
and would not indicate the precise injured branch of
the radial nerve. Both the deep branch of the ulnar
nerve and the median nerve traverse the medial and
anteromedial aspect of the arm, respectively. These
nerves primarily supply the fl exor compartment of
the arm. The anterior interosseous nerve is a branch
of the median nerve and supplies the fl exor digitorum
profundus, fl exor pollicis longus, and the pronator
quadratus.
GAS 750; GA 403

118
Q

A 34-year-old woman is admitted to the emergency
department after a car crash. Radiographic studies show
marked edema and hematoma of the arm, but there are
no fractures. During physical examination the patient
presents with inability to abduct her arm without fi rst
establishing lateral momentum of the limb, and inability
to fl ex the elbow and shoulder. Which of the following
portions of the brachial plexus is most likely injured?
⃣ A. Superior trunk
⃣ B. Middle trunk
⃣ C. Inferior trunk
⃣ D. Lateral cord
⃣ E. Medial cord

A

A. The superior trunk of the brachial plexus
includes C5 and C6, which give rise to the suprascapular
nerve, which innervates the supraspinatus
muscle. The supraspinatus muscle is the primary
muscle involved in abduction of the arm from 0 ° to
15 ° . The deltoid muscle, supplied primarily by C5,
abducts the arm from 15 degrees to 90 degrees. The
middle trunk is just C7 and has nothing to do with the
muscle involved in initial abduction of the arm. The
inferior trunk is C8-T1 and does not supply the supraspinatus
muscle; therefore, it is not the right answer.
The cords are distal to the branching of the supraspinatus
muscle; therefore, neither lateral cord nor medial
cord is the correct answer.
GAS 700-709; GA 361, 369-371

119
Q

A 22-year-old man is admitted to the hospital after
a car collision. Radiographic examination reveals an
oblique fracture of his humerus. Upon physical examination
the patient is unable to extend his forearm. The
damaged nerve was most likely composed of fi bers
from which of the following spinal levels?
⃣ A. C5, C6
⃣ B. C5, C6, C7
⃣ C. C5, C6, C7, C8, T1
⃣ D. C6, C7, C8, T1
⃣ E. C7, C8, T1

A

C. The radial nerve acts to extend the forearm at
the elbow. This nerve is derived from all the roots of
the brachial plexus C5 to T1. None of the other answers
include all the roots and are therefore incorrect.
GAS 700-709; GA 361, 369-371

120
Q

A 56-year-old woman is admitted to the hospital
after a severe car crash. A large portion of her chest wall
needed to be surgically removed and replaced with a
musculoosseous scapular graft involving the medial
border of the scapula. Which of the following arteries
will most likely recompensate the blood supply to the
entire scapula?
⃣ A. Suprascapular
⃣ B. Dorsal scapular artery
⃣ C. Posterior humeral circumfl ex artery
⃣ D. Lateral thoracic
⃣ E. Supreme thoracic artery

A

A. The suprascapular artery arises as a major
branch of the thyrocervical trunk from the subclavian
artery. It has rich anastomoses with the circum-
fl ex scapular artery and could provide essential
blood supply to the scapula. The dorsal scapular
artery would be lost with the graft. None of the other
vessels listed is in position to provide adequate supply
to the scapula.
GAS 696, 697; GA 366, 368

121
Q

A 56-year-old woman visits the emergency department
after falling on wet pavement. Radiographic
examination reveals osteoporosis and a Colles’ fracture.
Which of the following carpal bones are often fractured
or dislocated with a Colles’ fracture?
⃣ A. Triquetrum and scaphoid
⃣ B. Triquetrum and lunate
⃣ C. Scaphoid and lunate
⃣ D. Triquetrum, lunate, and scaphoid
⃣ E. Triquetrum and pisiform

A

C. The scaphoid and lunate carpal bones are in
closest articulation with the radius, which is fractured
in a Colles’ fracture; therefore, they would most likely
be disrupted or fractured. The other carpal bones
listed do not have direct contact with the radius and
have a more distal location; therefore, they would not
be as likely to be injured with a Colles’ fracture.
GAS 734; GA 392

122
Q

A 3-year-old girl is admitted to the emergency
department with severe pain. History taking reveals
that the girl was violently lifted by her raised arm by
her mother to prevent the girl from walking in front of
a moving car. Which of the following is most likely the
cause of the pain?
⃣ A. Compression of median nerve
⃣ B. Separation of the head of radius from its articulation
with trochlea of humerus
⃣ C. Separation of head of radius from its articulation
with ulna and capitulum of humerus
⃣ D. Separation of ulna from its articulation with
trochlea of humerus
⃣ E. Stretching of radial nerve as it passes behind
medial epicondyle of humerus

A

C. This type of dislocation is common in children
and results when the radius is dislocated and
slips out from the anular ligament, which holds it in
place, articulating with the ulna and the capitulum of
the humerus. In adults the anular ligament has a good
“grip” at the radial neck, but in young children the
radial head is not fully developed, leading to an indistinct
neck. Compression of the median nerve is not
likely due to its medial position in the cubital fossa.
The radius does not articulate with the trochlea of the
humerus; the ulna articulates at this position. The
ulna is not likely to be dislocated because it is more
stable than the radius, which has only the anular ligament
for its support. The radial nerve does not pass
behind the medial epicondyle; rather, the ulnar nerve
does this, so this is not the correct answer.
GAS 735; GA 391

123
Q

A 61-year-old man was hit by a cricket bat in the
midhumeral region of his left arm. Physical examination
reveals normal elbow motion; however, he could not
extend his wrist or his metacarpophalangeal joints and
he reported a loss of sensation on a small area of skin on the dorsum of the hand proximal to the fi rst two digits.
Radiographic examination reveals a hairline fracture of
the shaft of the humerus just distal to its midpoint.
Which of the following nerves is most likely injured?
⃣ A. Median
⃣ B. Ulnar
⃣ C. Radial
⃣ D. Musculocutaneous
⃣ E. Axillary

A

C. Injury to the radial nerve can be caused by
a blow to the midhumeral region since the nerve
winds around the shaft of the humerus. The symptoms
described include the loss of wrist and fi nger
extension and a loss of sensation in an area of skin
supplied by the radial nerve.
GAS 751; GA 380

124
Q

A 34-year-old man is admitted to the hospital after
a car collision. Radiographic examination reveals a
fracture at his wrist. Physical examination reveals paralysis
of the muscles that act to extend the interphalangeal
joints ( Fig. 6-10 ). Which of the following nerves
is most likely injured?
⃣ A. Ulnar
⃣ B. Recurrent branch of median
⃣ C. Radial
⃣ D. Musculocutaneous
⃣ E. Anterior interosseous

A

A. The ulnar nerve innervates the dorsal and
palmar interossei, which act to abduct and adduct
the fi ngers and assist the lumbricals in their actions
of fl exing the metacarpophalangeal joints and extending
the interphalangeal joints. The recurrent
branch of the median nerve innervates the thenar
muscle group that functions in the movement of the
thumb. The radial and musculocutaneous nerves do
not innervate any muscles in the hand. The anterior
interosseous innervates the fl exor pollicis longus and
the pronator quadratus.
GAS 770-771; GA 417

125
Q

A 45-year-old woman is admitted to the hospital
with neck pain. An MRI examination reveals a herniated
disk in the cervical region. Physical examination
reveals weak triceps brachii muscles. Which of the following
spinal nerves is most likely injured?
⃣ A. C5
⃣ B. C6
⃣ C. C7
⃣ D. C8
⃣ E. T1

A

C. The triceps brachii muscle is innervated by
the radial nerve (primarily C7), which comes off C5
to T1 spinal nerves. Because the patient’s only motor
defi cit involves the triceps brachii muscles, one can
rule out C5 and C6, which supply fi bers to the axillary,
musculocutaneous, and upper subscapular
nerves. Damage to either of these roots would result
in additional motor defi cits of the shoulder and fl exor
compartment of the arm. One can also rule out C8-T1
because these roots form the medial pectoral nerve
and the medial brachial and antebrachial cutaneous
nerves. Damage to these roots would result in loss of
pectoral muscle function and cutaneous sensation
over the medial surface of the upper limb.
GAS 700-709; GA 361, 369-371

126
Q

A 34-year-old woman is admitted to the hospital
after a car collision. Physical examination reveals a
mallet fi nger. Which of the following conditions is expected
to be present during radiographic examination?
⃣ A. A lesion of the ulnar nerve at the distal fl exor
crease of the wrist
⃣ B. A separation of the extension expansion over
the middle interphalangeal joint
⃣ C. Compression of the deep ulnar nerve by dislocation
of the lunate bone
⃣ D. Avulsion fracture of the dorsum of the distal
phalanx
⃣ E. Fracture of the fourth or fi fth metacarpal bone

A

D. Mallet fi nger, also known as baseball fi nger,
is a deformity in which the fi nger will be permanently
fl exed at the distal interphalangeal joint, due to avulsion
of the insertion of the extensor tendon at the
distal phalanx.
GAS 760; GA 392, 394, 422

127
Q

A 42-year-old woman is admitted to the hospital
with injury to the upper (superior) trunk of the brachial
plexus. The diagnosis is Erb-Duchenne palsy.
Which of the following conditions is expected to be
present during physical examination?
⃣ A. Winged scapula
⃣ B. Inability to laterally rotate the arm
⃣ C. Paralysis of intrinsic muscles of the hand
⃣ D. Paraesthesia in the medial aspect of the arm
⃣ E. Loss of sensation in the dorsum of the hand

A

B. Injury to the superior trunk of the brachial
plexus can damage nerve fi bers going to the suprascapular,
axillary, and musculocutaneous nerves. Damage
to the suprascapular and axillary nerves causes
impaired abduction and lateral rotation of the arm.
Damage to the musculocutaneous nerve causes impaired
fl exion of the forearm. A winged scapula would
be caused by damage to the long thoracic nerve. The
long thoracic nerve is formed from spinal cord levels
C5, C6, and C7, so the serratus anterior muscle would
be weakened from the damage to C5 and C6, but the
muscle would not be completely paralyzed. The intrinsic
muscles of the hand are innervated by the ulnar
nerve, which would most likely remain intact. Paraesthesia
in the medial aspect of the arm would be caused
by damage to the medial brachial cutaneous nerve (C8-
T1; inferior trunk). Loss of sensation on the dorsum of
the hand would be caused by damage to either the ulnar
or radial nerves (C6 to T1).
GAS 700-709; GA 361, 369-371

128
Q

A 41-year-old woman is admitted to the hospital
after a car crash. Radiographic examination reveals a
transverse fracture of the radius proximal to the attachment
of the pronator teres muscle. The proximal portion
of the radius is deviated laterally. Which of the
following muscles will most likely be responsible for
this deviation?
⃣ A. Pronator teres
⃣ B. Pronator quadratus
⃣ C. Brachialis
⃣ D. Supinator
⃣ E. Brachioradialis

A

D. The supinator muscle attaches to the radius
proximally and when fractured would cause a lateral
deviation. The pronator teres originates on the medial
epicondyle and coronoid process of the ulna and inserts
on the middle of the lateral side of the radius,
pulling the radius medially below the fracture. Pronator
quadratus originates on the anterior surface of the
distal ulna and inserts on the anterior surface of the
distal radius, pulling the radius medially. Brachioradialis
originates on the lateral supracondylar ridge of
the humerus and inserts at the base of the radial styloid
process, far below the fracture. Brachialis originates
in the lower anterior surface of the humerus
and inserts in the coronoid process and ulnar tuberosity,
hence not causing an action on the radius.
GAS 715; GA 379

129
Q

A 45-year-old woman is bitten by a dog on the lateral
side of her hand. Two days later the woman develops
fever and swollen lymph nodes. Which of the following
group of lymphatics will most likely be involved?
⃣ A. Central
⃣ B. Humeral
⃣ C. Pectoral
⃣ D. Subscapular
⃣ E. Parasternal

A

A. Lymph from the lateral side of the hand
drains directly into humeral (epitrochlear) nodes then
to the central (axillary) nodes. Pectoral nodes receive
lymph mainly from the anterior thoracic wall, including
most of the breast. Subscapular nodes receive
lymph from the posterior aspect of the thoracic wall
and scapular region. Parasternal nodes receive lymph
from the lower medial quadrant of the breast.
GAS 710; GA 381

130
Q

A 25-year-old woman is admitted to the emergency
department after a car collision. Radiographic
examination reveals a fracture at the spiral groove of
the humerus. A cast is placed, and 3 days later the
patient complains of severe pain over the length of her
Fig. 6-10
UPPER LIMB
189
arm. During physical examination the arm appears
swollen, pale, and cool. Radial pulse is absent, and any
movement of the arm causes severe pain. Which of the
following conditions will most likely characterize the
fi ndings of the physical examination?
⃣ A. Venous thrombosis
⃣ B. Thoracic outlet syndrome
⃣ C. Compartment syndrome
⃣ D. Raynaud’s disease
⃣ E. Injury of the radial nerve

A

C. Compartment syndrome is characterized by
increased pressure within a confi ned space by a fascial
compartment, which impairs blood supply, resulting
in paleness. Venous thrombosis would not cause
pain but could cause death from a pulmonary embolism.
Thoracic outlet syndrome affects nerves in the
brachial plexus and the subclavian artery and blood
vessels between the neck and the axilla, far above the
cast. Raynaud’s disease affects blood fl ow to the
limbs when they are exposed to temperature changes
or stress. The fracture at the radial groove probably
resulted in a radial nerve injury but would not be responsible
for these symptoms.
GAS 744; GA 381

131
Q

A 22-year-old woman is admitted to the hospital
after falling from a tree. Radiographic examination reveals
fractured pisiform and hamate bones. Which of
the following nerves will most likely be injured?
⃣ A. Median
⃣ B. Recurrent median
⃣ C. Radial
⃣ D. Anterior interosseous
⃣ E. Deep ulnar

A

E. The deep branch of the ulnar nerve arises
at the level of the pisiform bone and passes between
the pisiform and the hook of the hamate, hence the
ulnar is the nerve most likely to be injured in this
patient. The median nerve enters the forearm between
the humeral and ulnar heads of the pronator
teres muscle then becomes superfi cial near the
wrist. The recurrent median enters the palm through
the carpal tunnel. The radial nerve divides into superfi
cial and deep branches when it enters the cubital
fossa.
GAS 771; GA 417

132
Q

A 43-year-old man visits the outpatient clinic
with a painful shoulder. Physical examination reveals a
painful arc syndrome due to supraspinatus tendinopathy.
Which of the following conditions will be present
during physical examination?
⃣ A. Painful abduction 0 ° to 15 °
⃣ B. Painful abduction 0 ° to 140 °
⃣ C. Painful abduction 70 ° to 140 °
⃣ D. Painful abduction 15 ° to 140 °
⃣ E. Painful abduction 40 ° to 140 °

A

A. The supraspinatus initiates abduction of the
arm during the fi rst 15 ° of abduction; palpation of the
tendon during this phase would result in pain from a
tendinopathy of the supraspinatus.
GAS 678; GA 361, 364

133
Q

A 54-year-old woman is admitted to the hospital
after falling from a tree with an outstretched hand.
Radiographic examination reveals a wrist dislocation.
Which of the following carpal bones will most likely be
involved?
⃣ A. Scaphoid-lunate
⃣ B. Trapezoid-trapezium
⃣ C. Hamate-lunate
⃣ D. Pisiform-triquetrum
⃣ E. Hamate-capitate

A

A. The hallmark fracture caused by a fall on an
outstretched hand is a scaphoid-lunate fracture; the
scaphoid and lunate are the two wrist bones most
proximal to the styloid process of the radius. All the
other wrist bones are less likely to be affected by this
injury.
GAS 786; GA 392

134
Q

A 62-year-old man is admitted to the emergency
department after falling on wet pavement. Radiographic
examination reveals a carpometacarpal fracture at the
base of the thumb. What is the term applied to the described
fracture?
⃣ A. Colles fracture
⃣ B. Scaphoid fracture
⃣ C. Bennett’s fracture
⃣ D. Smith’s fracture
⃣ E. Boxer’s fracture

A

C. Bennett’s fracture is a carpometacarpal fracture
at the base of the thumb. Smith’s fracture is also
called a reverse Colles’ fracture and is caused when
the distal fragment of the radius angles forward.
Colles’ fracture is also called “silver fork deformity”
because the distal fragment is displaced posteriorly.
Boxer’s fractures of the necks of metacarpal bones are
fractures to the fi ngers. A scaphoid fracture would be
indicated by pain in the anatomical snuffbox.
GAS 756; GA 392

135
Q

A 23-year-old woman is participating in a dry
ski-slope competition. The woman is admitted to the
emergency department after falling and catching her
thumb in the matting. Radiographic and physical examinations
reveal rupture of the ulnar collateral ligament
of the metacarpophalangeal joint of the thumb.
The thumb is extremely painful and an injection of
lidocaine is performed. What is the most likely diagnosis
in this case?
⃣ A. Gamekeeper’s thumb
⃣ B. Scaphoid fracture
⃣ C. Bennett’s fracture
⃣ D. Smith’s fracture
⃣ E. Boxer’s fracture

A

A. Interestingly, “gamekeeper’s thumb” was
a term coined because this injury was most commonly
associated with Scottish gamekeepers who, it
is said, killed small animals such as rabbits by
breaking their necks between the ground and the
gamekeeper’s thumb and index fi nger. The resulting
valgus force on the abducted MCP joint caused injury
to the ulnar collateral ligament. These days this
injury is more commonly seen in skiers who land
awkwardly with their hand braced on a ski pole,
causing the valgus force on the thumb as is seen in
this patient. Whereas the term “skier thumb” is
sometimes used, “gamekeeper’s thumb” is still in
common usage. Bennett fracture is a fracture at the
base of the metacarpal of the thumb. Scaphoid fracture
occurs after a fall on an outstretched hand, involving
the scaphoid and lunate bone. Colles’ fracture
is also called silver fork deformity because the
distal fragment of the radius is displaced posteriorly.
Boxer’s fracture is a fracture of the necks of the second
and third (and sometimes the fi fth) metacarpals.
Smith’s fracture is also called a reverse Colles’
fracture and is caused when the distal radius is
fractured and the distal radial fragment is angled
forward.
GAS 755; GA 395

136
Q

A 54-year-old woman is found unconscious in
her car. She is admitted to the hospital, and during
physical examination she has absent biceps brachii
refl ex. What is the spinal level of the afferent component
of this refl ex?
⃣ A. C5
⃣ B. C6
⃣ C. C7
⃣ D. C8
⃣ E. T1

A

B. The biceps brachii refl ex involves C5 and C6
spinal nerves. C5 provides the motor component; C6
the afferent side of the refl ex arc.
GAS 700-709; GA 361, 369-371

137
Q

A 54-year-old woman is found unconscious in her
bed. She is admitted to the hospital, and during physical
examination she has absence of her brachioradialis
refl ex. The ventral ramus of which spinal nerve is responsible
for this refl ex?
⃣ A. C5
⃣ B. C6
⃣ C. C7
⃣ D. C8
⃣ E. T1

A

B. The brachioradialis refl ex is performed by
tapping the tendon of the brachioradialis muscle. The
refl ex involves spinal nerves C5, C6, and C7. The major
contribution is from C6.
GAS 700-709; GA 361, 369-371

138
Q

A 55-year-old woman is admitted to the emergency
department after a car crash. Physical examination
reveals severe pain in the fl exor muscles of the
forearm; fi xed fl exion position of the fi nger; and swelling,
cyanosis, and anesthesia of the fi ngers. Which of
the following is the most likely diagnosis?
⃣ A. Colles’ fracture
⃣ B. Scaphoid fracture
⃣ C. Bennett’s fracture
⃣ D. Volkmann’s ischemic contracture
⃣ E. Boxer’s fracture

A

D. Volkmann’s contracture is a fl exion deformity
of the fi ngers and sometimes the wrist from an
ischemic necrosis of the forearm fl exor muscles.
Bennett’s fracture is a fracture at the base of the metacarpal
of the thumb. Scaphoid fracture occurs after a
fall on an outstretched hand and involves the scaphoid
and lunate bones. Colles’ fracture is also called
silver fork deformity because the distal fragment of
the radius is displaced posteriorly. Boxer’s fracture is
a fracture of the necks of the second and third (and
sometimes the fi fth) metacarpals. Smith’s fracture is
also called a reverse Colles’ fracture and is caused
when the distal radius is fractured, with the radial
fragment angled forward.
GAS 762; GA 398

139
Q

A 62-year-old man visits the outpatient clinic
with pain in his hand after falling on the outstretched
hand. Radiographic examination reveals a fracture of
the pisiform bone and hematoma of the surrounding
area. Which of the following nerves will most likely be
affected?
⃣ A. Ulnar
⃣ B. Radial
⃣ C. Median
⃣ D. Deep ulnar
⃣ E. Deep radial

A

D. The ulnar nerve enters the forearm by passing
between the two heads of the fl exor carpi ulnaris
and descends between and innervates the fl exor carpi
ulnaris and fl exor digitorum profundus (medial half)
muscles. It enters the hand superfi cial to the fl exor
retinaculum and lateral to the pisiform bone, where it
is vulnerable to damage and provides the deep ulnar
branch. The deep branch of the radial nerve arises
proximally in the forearm.
GAS 771; GA 417

140
Q

A 32-year-old woman visits the outpatient clinic
after injuring her elbow falling from her bicycle. Physical
examination reveals a “benediction attitude” of the
hand with the index and long fi ngers extended and the
ring and little fi ngers fl exed. Which of the following is
the most likely diagnosis?
⃣ A. Injury to median and radial nerves
⃣ B. Injury to median nerve
⃣ C. Injury to radial and ulnar nerves
⃣ D. Injury to ulnar nerve
⃣ E. Injury to median ulnar and radial nerves

A

B. “Benediction attitude” of the hand with the
index and long fi ngers straight and the ring and little
fi ngers fl exed is caused by an injury to the median
nerve. The long fl exors of the digits are supplied by
the median nerve; the unopposed radial nerve and
deep ulnar nerve supply the extensors of the digits
1–3, causing them to be in the extended position.
Digits 4 and 5 are slightly fl exed, because the fl exors
of the PIP joints are supplied by the ulnar nerve.
GAS 724; GA 371, 376

141
Q

A 54-year-old man is admitted to the emergency
department with severe chest pain. Electrocardiographic
evaluation reveals a myocardial infarction. Due
to the severity of the infarction, a coronary artery bypass
surgery using a radial artery graft is proposed.
Which of the following tests should be performed during
physical examination prior to the bypass graft
operation?
⃣ A. Allen test
⃣ B. Triceps reflex
⃣ C. Tinel test
⃣ D. Brachioradialis reflex
⃣ E. Biceps reflex

A

A. The Allen test involves compression of the
radial and ulnar arteries at the wrist with the fi ngers
fl exed tightly. Pressure is then released on either vessel
successively to determine the degree of supply to the
hand by either vessel and the patency of the anastomoses
between them. The usefulness of the radial artery
for bypass can thereby be assessed. The other tests have
nothing to do with the patency of the radial artery.
GAS 770; GA 400

142
Q

A 34-year-old man visits the outpatient clinic
with a painful upper limb after a fall onto a concrete
fl oor. Physical examination reveals that the patient has
weak abduction and adduction of his fi ngers but has no
diffi culty in fl exing them. The patient also has decreased
sensation over the palmar surface of the fourth
and fi fth fi ngers. Which of the following diagnoses is
most likely?
⃣ A. Compression of the median nerve in the carpal
tunnel
⃣ B. Injury of the radial nerve from fractured humerus
in the radial tuberosity
⃣ C. Compression of the median nerve as it
passes between the two heads of the pronator
teres
⃣ D. Compression of the radial nerve from the supinator
⃣ E. Injury of the ulnar nerve by a fractured
pisiform

A

E. The ulnar nerve enters the hand superfi cial to
the fl exor retinaculum and lateral to the pisiform bone
and innervates all the interossei via the deep branch.
These muscles are responsible for adduction and abduction
of the fi ngers. Flexion of the fi ngers is spared
because the fl exor digitorum superfi cialis and most of
the fl exor digitorum profundus are innervated by the
median nerve, which is unaffected by this injury. Had
the median nerve been compressed in the carpal tunnel,
one would have diffi culty with motion of the
thumb as a result of a lack of innervation of the thenar
muscles. An injury of the radial nerve in the arm will
result in extension defi cit in the forearm and hand.
GAS 744; GA 417